BCA - Computer Organization & Architecture
BCA - Computer Organization & Architecture
Table of Contents: Each unit has a well-defined table of contents. For example: “1.1.1.
(a)” should be read as “Module 1. Unit 1. Topic 1. (Sub-topic a)” and 1.2.3. (iii) should
be read as “Module 1. Unit 2. Topic 3. (Sub-topic iii).
Aim: It refers to the overall goal that can be achieved by going through the unit.
Learning Outcomes: These are demonstrations of the learner’s skills and experience
sequences in learning, and refer to what you will be able to accomplish after going
through the unit.
Did You Know?: You will learn some interesting facts about a topic that will help you
improve your knowledge. A unit can also contain Quiz, Case Study, Critical Learning
Exercises, etc., as metacognitive scaffold for learning.
Summary: This includes brief statements or restatements of the main points of unit and
summing up of the knowledge chunks in the unit.
Activity: It actively involves you through various assignments related to direct application
of the knowledge gained from the unit. Activities can be both online and offline.
e-References: This is a list of online resources, including academic e-Books and journal
articles that provide reliable and accurate information on any topic.
Video Links: It has links to online videos that help you understand concepts from a
variety of online resources.
Author
Dr. Fazal Noorbasha
Director CDOE
C. Shanath Kumar
Instructional Designer
Nabina Das
Content Editor
Kalyani Kunchala
Project Manager
K. D. N. Lakshmi
Graphic Designer
K. Dinesh
Dr. Fazal Noorbasha holds multiple degrees, including B.Sc. in Electronics, M.Sc. in Electronics,
B.Tech. in ECE, and M.Tech. in AI (Artificial Intelligence). He obtained his Ph.D. in VLSI under the
esteemed guidance of Dr. Ashish Verma, Director In-charge at the Institute of Engineering and
Technology and Dr. Harishingh Gour, Central University, Sagar, M.P., India.
With extensive experience in guiding students, Dr. Fazal has supervised 30 B.Tech. (ECE)
and 35 M.Tech. (VLSI) projects. He has successfully mentored three Ph.D. scholars who have
completed their research work, and currently supervises three scholars who are pursuing their
Ph.D. under his guidance.
Dr. Fazal has an impressive publication record, with 57 papers indexed in Scopus/SCI/WoS, as
well as 30 journal and conference publications listed in UGC CARE. He has also undertaken
funded projects sponsored by DST and NGOs.In 2011,
Dr. Fazal joined K L University as an Assistant Professor in the Department of ECE. He initiated
the VLSI Research Group at the university, where he serves as the Research group head. Over
the years, he has organised numerous workshops, FDPs, and conferences as a convener and
co-convener. Presently, Dr. Fazal holds the position of Associate Professor in the Department of
ECE and serves as the Associate Dean (Academics) at K L University.
His research interests span across various domains, including Digital Systems, Cryptography,
VLSI, AI and IoT.
Course Description
The Computer Organisation and Architecture (COA) is one of the most important and comprehensive
subjects that includes many foundational concepts and knowledge used in the design of a computer
system. The system architecture defines various functional units of the computer system and how
these units are interconnected. It defines the system performance specifications and what system
should achieve in terms of performance.
In simple words, the computer architecture is all about computer system design details expressed in
terms of functional units and interconnection between these units. The computer architecture helps
us define the functional capabilities and the requirements for the computer system. The system
architecture is a high-level design specification that does not specify any specific details of the
hardware components. The computer architecture gives an abstracted view of the structure of various
functional units and its behaviour.
In Computer Organisation and Architecture (COA), the computer system can be classified into number
of functional units. This classification is based on the specific function performed in the computer
system.
Digital electronics, digital circuits, and digital technology are electronics that are operated on digital
signals. Digital techniques are much easier for getting the electronic device. These devices are used
to switch into one of the known states apart from reproducing a continuous range of values. Digital
circuits are made from an enormous collection of logic gates and a simple electronic representation
of the Boolean logic function. Modern digital computers are built from digital logic circuits whose basic
building blocks are logic gates, each of which is designed to implement a specific logical function.
Information is held in data “words”, representing data or instructions, made up from strings of individual
“bits” with binary values of 1 or 0. These values are analogous to Boolean logic propositions and
the statements or conclusions derived from them which were defined as “true” or “false”. Boolean
algebra is the tool used to design combinations of gates to implement more complex functions such
as mathematical operations, control functions and data storage.
Boolean functions are the building blocks of symmetric cryptographic systems. Symmetrical
cryptographic algorithms are fundamental tools in the design of all types of digital security systems
(i.e., communications, financial and e-commerce). Cryptographic Boolean Functions and Applications
is a concise reference that shows how Boolean functions are used in cryptography. Boolean functions
are among the fundamental objects of discrete mathematics, especially in those of its subdisciplines
which fall under mathematical logic and mathematical cybernetics.
With such wide applications, the presence of the digital electronics is felt in all the arenas of life and
carried further with new inventions. This course is designed for the aspirants who wish to know the
core concepts of Computer Organisation and Architecture using digital electronics concepts. It covers
the basic and academic concepts that include various computer blocks, number system, conversion
types, Logic Gates, SOP and POS Boolean equations and optimisation methods using Boolean laws
and K-MAPs, combinational logic circuits and sequential logic circuits design and many more.
MODULE 2
BOOLEAN ALGEBRA AND LOGIC GATES
Boolean Algebra - Boolean Algebra, Application of Boolean Algebra, Boolean Functions, Rules
of Boolean Algebra, Properties of Boolean Algebra and , Operations of Boolean Algebra- Boolean
Addition, Boolean Subtraction. Logic Gates, Truth Table and Logic Design – AND Gate, NAND Gate,
OR Gate, NOR Gate, EX-OR Gate, EX-NOR Gates.
MODULE 3
KARNAUGH MAP (K-MAP) AND SIMPLIFICATION OF BOOLEAN FUNCTIONS
Karnaugh Map-K-Map, Conversion Steps for Solving K-Map, steps used to solve the expressions,
Method of K-Map- 2 Variable KMaps, 3 Variable K Maps, 4 Variable K Maps, 5 Variable K Maps.
Minimization Techniques- Minterms and Maxterms, Sum of Product (SOP), Product of Sum (POS) and
Conversion of SOP to POS and Conversion of POS to SOP.
MODULE 4
COMBINATIONAL CIRCUITS & SEQUENTIAL CIRCUITS
Combinational Circuits-Half Adder, Full Adder, Half Subtractor, Full Subtractor, Multiplexer and
Demultiplexer. Conversion
Sequential Circuits– Flip Flops, Types of Flip Flop– SR Flip Flop, JK Flip Flop, D flip flop, T Flip flop,
Master - Slave JK Flip Flop, Conversion.
MODULE 1
BASICS OF COMPUTERS AND NUMBER SYSTEM
MODULE 2
BOOLEAN ALGEBRA AND LOGIC GATES
MODULE 3
KARNAUGH MAP (K-MAP) AND SIMPLIFICATION OF BOOLEAN FUNCTIONS
MODULE 4
COMBINATIONAL CIRCUITS & SEQUENTIAL CIRCUITS
Module - 1
Basics of Computers
and Number System
Now a days, computers have become an essential element in all aspects of life. Their importance lies
in all areas of the global needs that demand knowledge of computers as the basic need in making
an individual more updated to the global events. Therefore, knowledge of the basics of computers is
imperative for swift and expeditious work in all organisations.
Be it the latest developments, like Artificial Intelligence, Internet of Things, and Cloud Computing, etc.,
all the developments are the furtherance of the basic knowledge of the computers, and we cannot
deny the fact that the future runs on the miracles of the new innovations brought by the computers
and the future generations work environment is almost revolved around the computers and their
inventions. So, it is important for all of us to focus on the foundational knowledge of the computers and
their applications in different genres.
Globalisation has increased the scope of connectivity among all the nations with large amount of
data storing and sharing. With such enormous amounts of data connectivity, Big Data analytics
acquires importance on the parameters of memory allocation and division. The basic understanding
of the memory and the types acts as a basis to improvise the data processing and the storage of the
instructions from all the ends.
However, the functioning of the computer requires the knowledge of the number system for the efficient
flow of data. The encryption and decryption of data is important in the cyber arena which works on the
principle of number system to avoid any kind of data breach. Therefore, the basic understanding of the
number system gives ample scope to excel in data protection.
The basic knowledge of the computer includes the understanding of the components of the computer
input unit, CPU (Central Processing Unit), output unit, Memory, and Number System. Hence, this
module attempts to provide the basic knowledge on the above-mentioned topics.
Module - 1
Unit - 1
Basics of Computers
Aim --------------------------------------------------------------------------------------------------------- 08
Instructional Objectives ------------------------------------------------------------------------------- 08
Learning Outcomes ------------------------------------------------------------------------------------ 08
1.1.1 Central Processing Unit (CPU) -------------------------------------------------------------- 10
Self-Assessment Questions ------------------------------------------------------------------ 11
1.1.2 Control Unit --------------------------------------------------------------------------------------- 12
Self-Assessment Questions ------------------------------------------------------------------ 13
1.1.3Arithmetic Logic Unit ---------------------------------------------------------------------------- 14
Self-Assessment Questions ------------------------------------------------------------------ 15
1.1.4 Input Unit ------------------------------------------------------------------------------------------ 16
Self-Assessment Questions ------------------------------------------------------------------ 17
1.1.5 Output Unit ---------------------------------------------------------------------------------------- 18
Self-Assessment Questions ------------------------------------------------------------------ 19
1.1.6Memory or Storage Unit ------------------------------------------------------------------------ 20
Self-Assessment Questions ------------------------------------------------------------------ 21
Summary ------------------------------------------------------------------------------------------------- 22
Terminal Questions ----------------------------------------------------------------------------------- 22
Answer Keys -------------------------------------------------------------------------------------------- 23
Activity ---------------------------------------------------------------------------------------------------- 23
Glossary ------------------------------------------------------------------------------------------------ 24
Bibliography -------------------------------------------------------------------------------------------- 24
e-References ------------------------------------------------------------------------------------------ 24
Video links ---------------------------------------------------------------------------------------------- 25
Image credits ------------------------------------------------------------------------------------------ 25
Keywords ------------------------------------------------------------------------------------------------ 25
Instructional Objectives
This unit intends to:
Learning Outcomes
Upon completion of this unit, you will be able to:
Control Unit
Input Arithmetic/Logic Unit Output
Device Device
Memory Unit
Now, let’s delve into a detailed exploration of these functional components that comprise a computer
system.
The Von Neumann computer architecture, proposed by John von Neumann in the 1940s, is the foundation
for modern computer design. It features a sequential execution model with four key components: a
central processing unit (CPU), a memory unit, an input/output (I/O) system, and a control unit. The
CPU fetches and executes instructions stored in memory, while the control unit coordinates the flow
of data and instructions. This architecture enables the storage and manipulation of both program
instructions and data in a unified memory space, revolutionising the field of computing.
Computer Organisation & Architecture
9
1.1.1 Central Processing Unit (CPU)
A Central Processing Unit is also called a processor, central processor, or microprocessor. It carries
out all the essential functions of a computer. It receives instructions from both the hardware and active
software and produces output accordingly. It stores all important programs like operating systems and
application software. CPU also helps Input and output devices to communicate with each other. Owing
to these features of CPU, it is often referred to as the brain of the computer.
The CPU is physically installed or inserted into a CPU socket on the motherboard. To ensure optimal
performance, it is equipped with a heat sink that absorbs and dissipates heat, keeping the CPU cool.
The Central Processing Unit (CPU) encompasses the following features:
● Control Unit: The control unit is responsible for fetching and decoding instructions, as well as
coordinating the activities of other CPU components.
● Arithmetic Logic Unit (ALU): The ALU performs arithmetic and logical operations, including
addition, subtraction, comparison, and Boolean operations.
● Memory or Storage Unit: The memory unit stores data, intermediate results, and program
instructions.
A) CPU
B) Monitor
C) Mouse
D) Printer
3) CPU helps input and output devices to communicate with each other.
(True/False)
A) Execution
B) Runtime
C) Pipelining
D) Sequencing
5) Inside the CPU cabinet, the SATA and PATA cables are connected from motherboard to
______.
A) Hard disk
B) Floppy disk
C) ROM
D) RAM
The control unit is a crucial circuitry component within the computer system. It utilises electrical signals
to instruct the computer system in executing previously stored instructions. Its main responsibility is
to maintain and regulate the flow of information across the processor. Although the control unit does
not participate in processing and storing data, it plays a vital role in controlling and coordinating the
functioning of all computer parts.
During program execution, the control unit fetches one instruction at a time from the main memory,
decodes it, and then proceeds to execute it. If the instruction involves arithmetic or logical operations,
such as AND, OR, or Ex-OR, the control unit relies on the Arithmetic Logic Unit (ALU) to perform these
operations. After executing the current instruction, the CPU fetches the next instruction for further
execution. This process continues until the program is completed, and the result is output using an
output device. In many computer systems, the control unit and ALU are integrated into a single block
known as the Central Processing Unit (CPU).
The control unit carries out the following functions:
● It controls the transfer of data and instructions among the various units of the computer.
● It manages and coordinates all the units within the computer.
● It obtains instructions from the memory, interprets them, and directs the operation of the
computer accordingly.
● It facilitates communication with input/output devices to transfer data or store results.
● However, it does not process or store data itself.
7) ______ looks after and controls the working of the components and the computer system.
A) Control Unit
B) Monitor
C) RAM
D) All the above
8) Which one of the following helps the computer system in the process of carrying out the
stored program instructions:
A) ALU
B) Control Unit
C) Both
D) None
9) Control Unit interacts with both the main memory and ALU.
(True/False)
● Arithmetic Section: This section handles arithmetic operations such as addition, subtraction,
multiplication, and division. Although multiplication and division can be performed, they are
considered more resource-intensive operations. Multiplication can be achieved through
repetitive additions, while subtraction can be accomplished through repeated subtractions.
● Logic Section: This section focuses on logical operations, including AND, OR, NOT, XOR,
NOR, NAND, and other similar operations.
● Logical Operations: These involve logical operations such as AND, OR, NOT, XOR, NOR,
NAND, and others.
● Bit-Shifting Operations: This refers to shifting the positions of bits by a specific number of
places either to the right or left, resembling multiplication or division operations.
● Arithmetic Operations: These encompass bit-level addition and subtraction, with multiplication
and division being less frequently used due to their increased complexity. Multiplication can
be achieved through repeated additions, and subtraction can be performed through repeated
subtractions.
A) 2
B) 3
C) 4
D) 5
11) Which section is to perform logic operations such as comparing, selecting, matching, and
merging of data?
A) Arithmetic Section
B) Logic Section
C) Both A and B
D) None of the above
12) The ‘heart’ of the processor which performs many different operations:
13) ALU is the place where the actual executions of instructions take place during the pro-
cessing operation.
(True/False)
14) The ALU gives the output of the operations, and the output is stored in the _______.
A) Memory Devices
B) Registers
C) Flags
D) Output Unit
A) Remote
B) Microphone
C) Paddle
D) All the above
16) In computing, which one of the following is used to provide data and control signals to an
information processing system.
A) Control unit
B) Input unit
C) Output unit
D) None
17) The input device Keyboard has ________ keys on the topmost row.
A) Numeric keys
B) Function Keys
C) Control Keys
D) Special Keys
1. Monitor
● CRT Monitor
● LCD Monitor
● LED Monitor
● Plasma Monitor
2. Printer
● Impact Printers
A. Character Printers
● Dot Matrix printers
● Daisy Wheel printers
B. Line printers
● Drum printers
● Chain printers
● Non-impact printers
A. Laser printers
B. Inkjet printers
3. Projector
Monitors provide visual output, while printers produce hard copies of documents or images. Projectors
are used to display output on a larger screen or surface.
These output devices serve various purposes and offer different features to meet the diverse needs
of users. They play a crucial role in presenting the processed information to users in a readable,
printable, or viewable format.
18) Which one of the following converts the computer data to human understandable format?
19) The output displayed can be in diverse ways such as text, images and hardcopies.
(True/False)
20) Which one of the following allows users to project their output onto a large area, such as
a screen or a wall?
A) Projector
B) Speaker
C) Printer
D) None
The efficient functioning and capacity of the memory unit significantly influence the overall performance
of a computer system.
A) Paging
B) Segmentation
C) Bifurcation
D) Dynamic Division
A) Memory
B) Storage
C) Both
D) None
A) Primary Memory
B) Cache Memory
C) Both
D) None
24) Which unit can store instructions, data, and intermediate results?
Terminal Questions
Self-Assessment Questions
Question No. Answer
1 A
2 A
3 True
4 D
5 A
6 D
7 A
8 B
9 True
10 A
11 C
12 A
13 True
14 B
15 D
16 B
17 B
18 D
19 True
20 A
21 B
22 A
23 B
24 A
Activity
1. Draw a CPU block diagram, and then explain the function and interaction of each module
within the diagram.
Bibliography
● Mano, M. (1992). Computer System Architecture. PHI.
● Stallings, W. (2015). Computer System Architecture. PHI.
● Brown, S., & Vranesic, Z. (2008). Fundamentals of Digital Logic with Verilog Design (2nd
ed.). McGraw-Hill.
● Mano, M. (2017). Digital Logic and Computer Design. Pearson.
● Jain, R. P. (2009). Modern Digital Electronics (4th ed.). Tata McGraw Hill.
e-References
● https://www.tutorialspoint.com/computer_fundamentals/computer_cpu.htm
● https://www.tutorialspoint.com/computer_fundamentals/computer_input_devices.htm
● https://www.tutorialspoint.com/computer_fundamentals/computer_output_devices.htm
● https://study.com/academy/lesson/arithmetic-logic-unit-alu-definition-design-function.
html
● https://youtu.be/gHMQftD-HJY
Image Credits
● Fig.1: Wikipedia
Keywords
● Processor
● Input devices
● Output devices
● Arithmetic Operations
● Logical Operations
● Cache Memory
● HDD
● SSD
● DDR SDRAM
Module - 1
Unit - 2
Memory
Aim ----------------------------------------------------------------------------------------------------- 28
Instructional Objectives -------------------------------------------------------------------------- 28
Learning Outcomes -------------------------------------------------------------------------------- 28
1.2.1 Introduction to Memory -------------------------------------------------------------------- 29
1.2.1.1 Overview of Memory Unit ------------------------------------------------------ 29
1.2.2 Types of Memory ---------------------------------------------------------------------------- 31
1.2.2.1 Random Access Memory ------------------------------------------------------- 31
1.2.2.2 Read-Only Memory --------------------------------------------------------------- 32
Self-Assessment Questions -------------------------------------------------------------- 35
Summary --------------------------------------------------------------------------------------------- 39
Terminal Questions ------------------------------------------------------------------------------- 39
Answer Keys ---------------------------------------------------------------------------------------- 40
Activity ----------------------------------------------------------------------------------------------- 40
Glossary --------------------------------------------------------------------------------------------- 41
Bibliography ----------------------------------------------------------------------------------------- 41
e-References --------------------------------------------------------------------------------------- 42
Video Links ------------------------------------------------------------------------------------------ 42
Image Credits ---------------------------------------------------------------------------------------- 42
Keywords --------------------------------------------------------------------------------------------- 42
Instructional Objectives
This unit intends to:
Learning Outcomes
Upon completion of this unit, you will be able to:
2. Functionality:
Memory units are responsible for performing three key functions:
● Data Storage: Memory units store data in binary form, consisting of 0s and 1s, which represent
the fundamental building blocks of information within a computer system. This data can include
program instructions, input data, intermediate results, and output data generated during
computational processes.
● Data Retrieval: The CPU interacts with the memory unit to retrieve stored data and instructions
required for processing. The memory unit ensures quick access and retrieval of information,
enabling the CPU to execute instructions efficiently.
● Data Manipulation: The memory unit facilitates data manipulation by the CPU. It allows the
CPU to read, write, and modify data stored within its memory cells, providing the necessary
workspace for computations and operations.
address of address of
memory cell RAM ( memory ) memory cell RAM ( memory )
13
00000000 00001101
3
00000001 00000011
0
00000010 00000000
45
00000011 00101101
Types of memory
RAM ROM
Random Access Memory, commonly known as RAM, is a type of volatile memory that allows data to be
read from and written to by the computer’s processor. It provides fast access to data and instructions
during program execution.
1. Static RAM (SRAM): Static RAM is a type of RAM that uses flip-flop circuits to store data. It is
faster and more expensive than other types of RAM. SRAM is commonly used in cache memory, which
provides quick access to frequently used data.
2. Dynamic RAM (DRAM): Dynamic RAM stores data using capacitors, which need to be refreshed
periodically to retain the data. DRAM is more cost-effective but slower compared to SRAM. It is widely
used as the main memory in computers due to its higher storage capacity.
● Speed: RAM allows quick access to data compared to other storage types.
● Flexibility: Data stored in RAM can be easily modified or deleted.
● Capacity: RAM capacity can be upgraded to store more data, improving performance.
● Power Management: RAM consumes less power, making it suitable for portable devices.
● Volatility: Data stored in RAM is lost when power is turned off, posing a risk to unsaved work.
● Capacity: RAM has limitations and may not be sufficient for memory-intensive tasks.
● Cost: Upgrading RAM can be relatively expensive compared to other memory types.
Read-Only Memory, or ROM, is a non-volatile memory that stores data and instructions that cannot be
modified or erased by normal computer operations. It contains firmware and system-specific software
required for booting the computer.
● Stores crucial information essential to operate the system, such as the program required to boot
the computer.
● It is non-volatile, meaning it retains its data even when power is turned off.
● Used in embedded systems or applications where the programming does not need to be
changed.
● Commonly found in calculators and peripheral devices.
1. PROM (Programmable Read-Only Memory): Can be programmed by the user, but the data and
instructions in it cannot be changed once programmed.
2. EPROM (Erasable Programmable Read Only Memory): Can be reprogrammed by erasing data
through exposure to ultraviolet light. All previous data must be erased before reprogramming.
● Negative charges form a channel between the source and drain, storing logic 1.
● Applying a large positive voltage at the gate causes negative charges to move out of the channel
and get trapped in the floating gate, storing logic 0.
● To erase the EPROM, UV rays are shone on the surface of the floating gate, causing the
negative charges to return to the channel from the floating gate, restoring logic 1.
● EPROM packages have a quartz window through which UV light can pass, enabling the erasure
process.
3. EEPROM (Electrically Erasable Programmable Read Only Memory): The data can be erased
by applying an electric field, allowing selective erasure of portions of the chip. Unlike EPROM, which
requires the entire chip to be erased, EEPROM allows for selective erasure of portions of the chip.
EEPROM is programmed and erased electronically, typically by utilising higher than normal voltage
levels. Some key features of EEPROM include:
● It offers improved writeability, allowing data to be written or modified more easily compared to
other types of memory.
● It can be in-system programmable with built-in circuit to provide the necessary higher voltage
levels.
● It has built-in memory controller, which is commonly used to hide complex details from the
memory user, simplifying the interaction with the memory.
● Due to the erasing and programming processes involved, EEPROM tends to have slower write
speeds compared to other memory types.
● It can endure tens of thousands of erase and program cycles, allowing for data modification
without degradation.
● Shares a similar storage permanence characteristic with EPROM, with data retention typically
lasting around 10 years.
● More convenient to use than EPROM due to the absence of ultraviolet light for erasure. However,
it is generally more expensive.
● EEPROM erases data in 8-bit units, utilising high electrical voltage.
4. MROM (Mask ROM): A type of ROM that is permanently programmed during the manufacturing
process and cannot be modified by the user.
● Non-volatility: ROM retains data even when power is turned off, making it suitable for storing
critical data like BIOS or firmware.
● Reliability: The data in ROM is less prone to corruption or errors compared to other types of
memory.
● Power Management: ROM consumes less power, making it suitable for portable devices.
● Limited Flexibility: ROM is read-only and cannot be modified, which can be problematic for
applications or firmware that require updates.
● Limited Capacity: ROM typically has limited capacity and upgrading it can be difficult or expen-
sive.
● Cost: ROM can be more expensive than other types of memory, impacting the cost of upgrading
computer systems or devices.
3 Volatile Non-volatile
1) The chip by which both the operation of read and write is performed __________.
A) RAM
B) ROM
C) PROM
D) EPROM
A) RWM
B) MBR
C) MAR
D) ROM
3) Which of the following control signals are selected for read and write operations in a RAM?
A) Data buffer
B) Chip select
C) Read and write
D) Memory
A) High complexity
B) High resolution
C) High speed main memory
D) High flexibility
5) Which of the following has the capability to store the information permanently?
A) RAM
B) ROM
C) Storage cells
D) Both RAM and ROM
A) low-cost
B) high-cost
C) low power consumption
D) transistor as a storage element
9) Which type of storage element of SRAM is extremely fast in accessing data but con-
sumes lots of power?
A) TTL
B) CMOS
C) NAND
D) NOR
A) 4ns
B) 10ns
C) 2ns
D) 60ns
A) 2
B) 4
C) 6
D) 8
12) Which memory storage is widely used in PCs and Embedded Systems?
A) SRAM
B) DRAM
C) Flash memory
D) EEPROM
A) Inductor
B) Capacitor
C) Resistor
D) Mosfet
A) 1 Mb
B) 4-256 Mb
C) 8-128 Mb
D) 64-128 Mb
A) ROM
B) EPROM
C) PROM
D) EEPROM
18) Address decoding for dynamic memory chip control may also be used for ____________.
19) Which one of the following is used for the fabrication of MOS EPROM?
A) TMS 2513
B) TMS 2515
C) TMS 2516
D) TMS 2518
A) 1024
B) 512
C) 2516
D) 256
Terminal Questions
Self-Assessment Questions
Question No. Answer
1 A
2 A
3 C
4 C
5 B
6 C
7 B
8 C
9 A
10 A
11 C
12 B
13 B
14 B
15 C
16 C
17 B
18 A
19 C
20 C
Activity
Bibliography
● Mano, M. (1992). Computer System Architecture. PHI.
● Brown, S., & Vranesic, Z. (2008). Fundamentals of Digital Logic with Verilog Design (2nd
ed.). McGraw-Hill.
● Jain, R. P. (2009). Modern Digital Electronics (4th ed.). Tata McGraw Hill.
Video links
● https://youtu.be/nSEfOe88Y0w
● https://youtu.be/sWAsRA-9DgQ
● https://youtu.be/CPOcSGgSxiQ
Image credits
● Fig.1: https://passlab.github.io/ITSC3181/notes/lecture04_MemoryBinarySystem.pdf
● Fig.2: https://media.geeksforgeeks.org/wp-content/uploads/memory.png
Keywords
● RAM
● ROM
● SRAM
● DRAM
● PROM
● EEPROM
● MOSFET
Module - 1
Unit - 3
Number System
Aim --------------------------------------------------------------------------------------------- 45
Instructional Objectives ------------------------------------------------------------------ 45
Learning Outcomes ------------------------------------------------------------------------ 45
1.3.1 Overview of Number System ---------------------------------------------------- 46
1.3.2 Types of Number Systems ------------------------------------------------------- 47
Self-Assessment Questions ----------------------------------------------------- 50
1.3.3 Conversion of Number Systems ------------------------------------------------ 51
1.3.4 Conversion of Decimal Number System -------------------------------------- 51
Self-Assessment Questions ----------------------------------------------------- 58
1.3.5 Conversion of Binary Number System ---------------------------------------- 59
Self-Assessment Questions ----------------------------------------------------- 64
1.3.6 Conversion of Octal Number System ------------------------------------------ 65
Self-Assessment Questions ----------------------------------------------------- 69
1.3.7 Conversion of Hexadecimal Number System ------------------------------- 70
Self-Assessment Questions ----------------------------------------------------- 74
Summary ------------------------------------------------------------------------------------ 75
Terminal Questions ----------------------------------------------------------------------- 75
Answer Keys -------------------------------------------------------------------------------- 76
Activity -------------------------------------------------------------------------------------- 76
Glossary ------------------------------------------------------------------------------------ 77
Bibliography -------------------------------------------------------------------------------- 77
e-References ------------------------------------------------------------------------------ 78
Video Links --------------------------------------------------------------------------------- 78
Image Credits ----------------------------------------------------------------------------- 78
Keywords ------------------------------------------------------------------------------------ 78
Instructional Objectives
This unit intends to:
Learning Outcomes
Upon completion of this unit, you will be able to:
Radix Point
● Binary Number System: Uses a radix of 2 and consists of only two digits, 0 and 1.
● Octal Number System: Uses a radix of 8 and consists of digits from 0 to 7.
● Decimal Number System: Uses a radix of 10 and consists of digits from 0 to 9.
● Hexadecimal Number System: Uses a radix of 16 and consists of digits from 0 to 9, along with
additional symbols A to F to represent values 10 to 15.
101 . 01
Binary Point
In the Binary Number System, the portion to the left of the binary point (integer part) has weights of
20, 21, 22, and so on from right to left. The portion on the right-hand side of the binary point (fractional
part) has weights of 2-1, 2-2, and so on from left to right.
The given binary number (101.01)2 can be represented as:
(101.01)2 = (1 * 22) + (0 * 21) + (1 * 20) + (0 * 2-1) + (1 * 2-2)
By simplifying the terms on the right-hand side of the equation, we obtain the decimal equivalent of the
binary number on the left-hand side, which is (5.25)10.
125 . 01
Octal Point
540 . 25
Decimal Point
Fig. 4: Decimal number representation format
In the Decimal Number System, the positional values or weights to the left of the decimal point in the
integer part are assigned as 100, 101, 102, and so on, from right to left. For the fractional part on the
right-hand side, the weights are assigned as 10-1, 10-2, and so on, from left to right.
It consists of both digits and alphabets where digits range from 0 to 9 i.e., 0, 1, 2, 3, 4, 5, 6, 7, 8, 9 and
alphabets range from A to F where (A=10), (B=11), (C=12), (D=13), (E=14) and (F=15).
12A . 13
Hexadecimal Point
In the Hexadecimal Number System, the positional values or weights to the left of the decimal point
in the integer part are assigned as 160, 161, 162, and so on, from right to left. For the fractional part on
the right-hand side, the weights are assigned as 16-1, 16-2, and so on, from left to right.
A) 0.34
B) 3.4
C) 34
D) 340
A) 11111111
B) 11111110
C) 00001111
D) 10000001
A) 00110011
B) 00110100
C) 00110101
D) 00110110
Solution:
Integral Part:
Divide the decimal integer part (13) by 2 successively until the quotient becomes 0:
13 ÷ 2 = 6 remainder 1
6 ÷ 2 = 3 remainder 0
3 ÷ 2 = 1 remainder 1
1 ÷ 2 = 0 remainder 1
Fractional Part:
Multiply the decimal fractional part (0.25) by 2 successively until the fractional part becomes 0:
0.25 × 2 = 0.5
0.5 × 2 = 1.0 (Note: Since the fractional part becomes 0, we stop here.)
Reading the integers obtained from top to bottom, we have: (0.25)10 = (0.01)2
(13.25)10 = (1101.01)2
Solution:
Integral Part:
Divide the decimal integer part (15) by 2 successively until the quotient becomes 0:
15 ÷ 2 = 7 remainder 1
7 ÷ 2 = 3 remainder 1
3 ÷ 2 = 1 remainder 1
1 ÷ 2 = 0 remainder 1
Fractional Part:
Multiply the decimal fractional part (0.6) by 2 successively until the fractional part becomes 0:
(Note: Since the fractional part repeats the sequence 1100, we can stop here.)
Reading the integers obtained, we have: (0.6)10 = (0.1 1001 1001...)2 (repeating)
Combining the integral and fractional parts, we get:
(15.6)10 = (1111.1100 1100...)2 (repeating)
(or)
(15.6)10 = (1111.1100)2
Therefore, (15.6)10 is equivalent to (1111.1100 1100...)2 (repeating) in binary.
Solution:
To convert the decimal number (73.625)10 to octal, we can follow the pro-
cess of separating the integral and fractional parts and converting them individually.
Integral Part:
We divide the integral part, 73, by 8 successively until the quotient becomes 0:
73 ÷ 8 = 9 remainder 1
9 ÷ 8 = 1 remainder 1
1 ÷ 8 = 0 remainder 1
Reading the remainders from bottom to top, we have the octal equivalent of the integral part: (111)8.
Fractional Part:
For the fractional part, 0.625, we multiply it by 8 successively until the fractional part of the product
becomes 0:
0.625 × 8 = 5
Solution:
To convert the decimal number (965.198)10 to octal, we can follow the process of separating the inte-
gral and fractional parts and converting them individually.
Integral Part:
We divide the integral part, 965, by 8 successively until the quotient becomes 0:
Fractional Part:
For the fractional part, 0.198, we multiply it by 8 successively until the fractional part of the product
becomes 0:
0.198 × 8 = 1.584
0.584 × 8 = 4.672
0.672 × 8 = 5.376
0.376 × 8 = 3.008
0.008 × 8 = 0.064
The integer parts of the product terms, read from top to bottom, represent the octal equivalent of the
fractional part: (14535)8.
Combining the integral and fractional parts, we get the final octal representation: (965.198)10 =
(1755.14535)8.
Solution:
Given decimal number (1954.785)10 is of mixed type and contains both integral (1954)10 and decimal
part (0.785)10. To convert the given number into hexadecimal, we have to convert integral and frac-
tional part individually into hexadecimal and then combine them together to get the required result.
Integral Part:
We successively divide the integral part by 16 until the quotient is 0 and record the remainders from
bottom to top.
1954 ÷ 16 = 122 remainder 2
122 ÷ 16 = 7 remainder 10 (A in hexadecimal)
The remainders are 10 and 2, which correspond to A and 2 in hexadecimal, respectively. So, the inte-
gral part in hexadecimal is A2.
Fractional Part:
For the fractional part, we multiply it by 16 until the fractional part of the product becomes 0. We record
the integer parts of the product terms from top to bottom.
0.785 × 16 = 12.56 (integer part: 12)
0.56 × 16 = 8.96 (integer part: 8)
0.96 × 16 = 15.36 (integer part: 15)
The integer parts are 12, 8, and 15, which correspond to C, 8, and F in hexadecimal, respectively. So,
the fractional part in hexadecimal is (C8F)16.
After converting both integral part and fractional part individually into hexadecimal, now we combine
both to get our desired result i.e., (1954.785)10 = (A2.C8F)16.
Solution:
To convert the given number into hexadecimal, we have to convert integral and fractional part indi-
vidually into hexadecimal and then combine them together to get the required result.
Integral Part:
We successively divide the integral part by 16 until the quotient is 0 and record the remainders from
bottom to top.
The remainders are 13 and 3, which correspond to D and 3 in hexadecimal, respectively. So, the
integral part in hexadecimal is D3.
Fractional Part:
For the fractional part, we multiply it by 16 until the fractional part of the product becomes 0. We
record the integer parts of the product terms from top to bottom.
The integer parts are 11, 7, and 0, which correspond to B, 7, and 0 in hexadecimal, respectively. So,
the fractional part in hexadecimal is B70.
Combining the integral and fractional parts, we get the hexadecimal representation of (3283.715)10
as (D3.B70)16.
A) 10101
B) 10001
C) 10000
D) 11111
5) The decimal number + 122 is expressed in the 2’s complement form as ______.
A) 01111010
B) 11111010
C) 01000101
D) 10000101
A) (FD)16
B) (DF)16
C) (AA)16
D) (AF)16
A) (0.16050)8
B) (0.26050)8
C) (0.19450)8
D) (0.24040)8
A) (641)8
B) (619)8
C) (640)8
D) (598)8
● In an integral part of the binary number, the positional weights follow the pattern of 20, 21, 22,
23, 24, 25, and so on from right to left.
● In the fractional part of the binary number, the positional weights follow the pattern of 2-1, 2-2,
2-3, 2-4, 2-5, and so on from left to right.
To convert a mixed binary number, we convert its integral and fractional parts individually and then
combine them to obtain the decimal number.
Let’s consider some examples to illustrate the concept more effectively.
Solution:
To convert the binary number (101.11)2 to its decimal equivalent, we use the positional weights and
perform the necessary calculations.
In the integral part, we have:
(101) 2 = (1 * 2²) + (0 * 2¹) + (1 * 2⁰) = 4 + 0 + 1 = 5.
In the fractional part, we have:
(0.11) 2 = (1 * 2-1) + (1 * 2-2) = 0.5 + 0.25 = 0.75.
Solution:
In the integral part, we have:
(11011) 2 = (1 * 2⁴) + (1 * 2³) + (0 * 2²) + (1 * 2¹) + (1 * 2⁰) = 16 + 8 + 0 + 2 + 1 = 27.
In the fractional part, we have:
(0.101) 2 = (1 * 2-1) + (0 * 2-2) + (1 * 2-3) = 0.5 + 0 + 0.125 = 0.625.
Solution:
To convert the binary number (110011.011) 2 to octal, we will separate the integral and fractional parts
and convert them individually.
Integral Part:
Starting from the rightmost side, we group the bits in sets of three: 110 011. Since we have three
groups, we can directly convert them to their octal equivalents: (6 3)8.
Solution:
To convert the binary number (110011011110.1011)2 to octal, we will separate the integral and fraction-
al parts and convert them individually.
Integral Part:
Starting from the rightmost side, we group the bits in sets of three: 110 011 011 110. We can then
convert each group to its octal equivalent: (6 3 3 6) 8.
Fractional Part:
For the fractional part, we start from the left and group the bits in sets of three: 101 100. We can then
convert it to its octal equivalent: (5 4)8.
Combining the integral and fractional parts, we have (110011011110.1011)2 = (6336.54)8.
Note: In the Example 2, to make a group of three bits, we have added two additional bits to the right
of LSB in the fractional part.
Converting binary numbers into hexadecimal numbers follows a similar process as converting binary
to octal, but with some modifications. The relationship between binary numbers and hexadecimal
numbers is given as:
In hexadecimal number system, we have sixteen digits ranging from 0 to 15 which can be represented
using four-bit binary numbers in 24 = 16 ways, so starting from the least significant bit of the binary
number, we group four successive bits of the binary number to get its equivalent hexadecimal number
as seen from the table above.
In the integral part, we group four bits from right to left, and in the fractional part, we group four bits
from left to right, and then convert them to their respective hexadecimal symbols. In the process of
grouping four bits, one/two/three bits can be added to the left of the MSB in an integral part and/or to
the right of the LSB bit of the fractional part of the binary number.
Note: Whenever we need any additional bits, we only add ‘0’ as the additional bit.
Let’s consider some examples to illustrate the concept more effectively.
Solution:
To convert the binary number to hexadecimal, we will separate the integral and fractional parts and
convert them individually.
Integral Part:
Group the binary digits in sets of four from left to right: “0111” and “1111”.
Convert each group to its decimal equivalent: “0111” = 7, “1111” = 15.
In hexadecimal, the decimal values 7 and 15 are represented as “7” and “F” respectively.
Fractional Part:
Consider the fractional part “1010”.
Solution:
To convert the given binary number to hexadecimal, we need to group the binary digits into sets of
four from left to right, both in the integral and fractional parts. However, in this case, the given binary
number consists of only 10 bits in the integral part and 6 bits in the fractional part, which cannot be
evenly grouped into sets of four.
To accommodate the grouping, we add two zero bits to the left of the most significant bit (MSB) in the
integral part, resulting in a 12-bit integral part. Similarly, we add two zero bits to the right of the least
significant bit (LSB) in the fractional part, resulting in an 8-bit fractional part.
The modified binary number is now: (001110011100.11000100)2
Now, we can group the binary digits into sets of four:
Integral part: 0011 1001 1100
Fractional part: 1100 0100
Converting each set of four binary digits to their hexadecimal equivalents, we get:
Integral part: 3 9 C
Fractional part: C 4
A) 2
B) 4
C) 6
D) 8
A) 121
B) 221
C) 441
D) 256
A) (11.6875)10
B) (11.5874)10
C) (10.9876)10
D) (10.7893)10
A) (5512)8
B) (6612)8
C) (4532)8
D) (6745)8
By replacing each digit with its binary equivalent, we can obtain the binary representation of the octal
number.
Let’s consider some examples to illustrate the concept more effectively.
Solution:
To convert the octal number (73.2)8 to binary, we need to convert each digit of the octal number to its
binary equivalent.
Solution:
To convert the octal number (475.62)8 to binary, we need to convert each digit of the octal number to
its binary equivalent.
Starting with the integral part, we have:
Digit 4: 4 in binary is 100.
Digit 7: 7 in binary is 111.
Digit 5: 5 in binary is 101.
Combining them, we get the integral part in binary as 100111101.
Solution:
To convert the octal number (75.3)8 to its decimal equivalent, we use the positional weights and multi-
ply each digit by the corresponding weight.
In the integral part, the positional weights follow the pattern of powers of 8: 80, 81, 82, and so on, from
right to left. In this case, we have the digits 7 and 5 in the integral part.
(7 * 81) + (5 * 80) = 56 + 5 = 61
Shortcut:
= 56+5+0.375
= (61.375)10
Solution:
= (404.894)10
Solution:
The given number (482.31)8 is not a valid octal number because octal numbers only consist of digits
ranging from 0 to 7. As the given number includes the digit 8, it cannot be converted to a decimal
number using octal-to-decimal conversion.
Solution:
Note: To know more about converting binary number to a hexadecimal number, refer to the topic “Con-
version of Binary number to Hexadecimal number.”
Solution:
A) (111101)2
B) (010100)2
C) (111100)2
D) (101010)2
A) (85)10
B) (95)10
C) (75)10
D) (65)10
A) (011101010110101011)2
B) (00111101010101001)2
C) (001101101010111001)2
D) (001110011011110001)2
Solution:
To convert the hexadecimal number to binary, we replace each hexadecimal digit with its equivalent
binary representation.
Using the table provided above, we can replace hexadecimal numbers with their equivalent binary
digits.
Solution:
Using the table provided above, we can replace hexadecimal numbers with their equivalent binary
digits.
Solution:
Using the table provided above, we can replace hexadecimal numbers with their equivalent binary
digits.
Conversion of the hexadecimal numbers to octal numbers follows a specific process. We first convert
the hexadecimal number to a binary number, and then convert the binary number to an octal number.
The conversion path is as follows: Hexadecimal Number → Binary Number → Octal Number
Solution:
(00011101.1110)2 = (35.7)8
Solution:
To convert a hexadecimal number into a decimal number, we use the positional weights by multiplying
each digit with the corresponding weight and summing them up.
● In the integral part of the hexadecimal number, the weights follow the pattern as 160, 161, 162,
163, 164, 165, and so on from right to left.
● In the fractional part of the hexadecimal number, the weights follow the pattern as 16-1, 16-2,
16-3, 16-4, 16-5, and so on from left to right.
It’s important to note that in hexadecimal representation, A = 10, B = 11, C = 12, D = 13, E = 14, and
F = 15.
By calculating the products and summing them up, we can obtain the decimal equivalent of the hexa-
decimal number.
Solution:
Step 1: Convert the integral part of the hexadecimal number to decimal.
(75)16 = (7 * 161) + (5 * 160) = (117)10
Step 2: Convert the fractional part of the hexadecimal number to decimal.
(0.3)16 = (3 * 16-1) = (0.1875)10
Combine the integral and fractional parts to obtain the decimal equivalent.
(75.3)16 = (117)10 + (0.1875)10 = (117.1875)10
Solution:
Step 1: Convert the integral part of the hexadecimal number to decimal.
(CD3)16 = (12 * 162) + (13 * 161) + (3 * 160) = (3283)10
Step 2: Convert the fractional part of the hexadecimal number to decimal.
(B70A)16 = (11 * 16-1) + (7 * 16-2) + (0 * 16-3) + (10 * 16-4) = (0.718994)10
Combine the integral and fractional parts to obtain the decimal equivalent.
(CD3.B70A)16 = (3283)10 + (0.718994)10 = (3283.718994)10
A) (35.684)8
B) (36.246)8
C) (34.340)8
D) (35.599)8
A) 480
B) 483
C) 482
D) 484
A) (711)10
B) (6781)10
C) (19999)10
D) (1019)10
Terminal Questions
1. Convert the Binary number (101010.1011)2 into Decimal, Hexadecimal and Octal.
4. Convert the Decimal number (235.92)10 into Octal, Hexadecimal, and Binary.
Self-Assessment Questions
Question No. Answer
1 C
2 C
3 B
4 A
5 A
6 C
7 B
8 A
9 D
10 B
11 A
12 B
13 B
14 A
15 A
16 B
17 C
18 C
Activity
Activity Type: Offline Duration: (30-40 min)
Four types of number system using the following data: (23.54)8, (93.54)10, (2A.54)16,
(101101.1011)2
Bibliography
● Mano, M. (1992). Computer System Architecture. PHI.
● Stallings, W. (2015). Computer System Architecture. PHI.
● Brown, S., & Vranesic, Z. (2008). Fundamentals of Digital Logic with Verilog Design (2nd
ed.). McGraw-Hill.
● Mano, M. (2017). Digital Logic and Computer Design. Pearson.
● Jain, R. P. (2009). Modern Digital Electronics (4th ed.). Tata McGraw Hill.
Video links
● https://youtu.be/crSGS1uBSNQ
● https://youtu.be/4ae9sJBBkvw
Image Credits
Keywords
● Base
● Numeral
● Radix Point
● Positional Notation
● Complement
● Arithmetic Operations
● Floating-Point Representation
● Overflow and Underflow
● Sign Bit
● Binary Coded Decimal (BCD)
Module - 2
Unit - 1
Aim ------------------------------------------------------------------------------------------------------- 82
Instructional Objectives ----------------------------------------------------------------------------- 82
Learning Outcomes --------------------------------------------------------------------------------- 82
2.1.1 Boolean Algebra, Rules, and Properties of Boolean Algebra --------------------- 83
Self-Assessment Questions --------------------------------------------------------------- 86
2.1.2 Boolean Functions --------------------------------------------------------------------------- 87
Self-Assessment Questions --------------------------------------------------------------- 89
2.1.3 De Morgan’s Theorem ---------------------------------------------------------------------- 90
Self-Assessment Questions --------------------------------------------------------------- 94
Summary ----------------------------------------------------------------------------------------------- 95
Terminal Questions ---------------------------------------------------------------------------------- 95
Answer Keys ----------------------------------------------------------------------------------------- 96
Activity ------------------------------------------------------------------------------------------------- 96
Glossary ----------------------------------------------------------------------------------------------- 97
Bibliography ----------------------------------------------------------------------------------------- 97
e-References ---------------------------------------------------------------------------------------- 97
Video Links ------------------------------------------------------------------------------------------- 97
Image Credits --------------------------------------------------------------------------------------- 97
Keywords --------------------------------------------------------------------------------------------- 98
Instructional Objectives
This unit intends to:
Learning Outcomes
Upon completion of this unit, you will be able to:
DID The Boolean algebra is used for simplifying and analysing the complex
YOU Boolean expression. It is also known as Binary algebra because we only
KNOW use binary numbers in this. George Boole developed the binary algebra
in 1854.
2. The overbar (-) is used for representing the complement variable. So, the complement of vari-
able C is represented as.
3. The plus (+) operator is used to represent the ORing of the variables.
1) Annulment Law
When the variable is AND with 0, it will give the result 0, and when the variable is OR with 1, it will
give the result 1, i.e.,
B.0 = 0
B+1 = 1
2) Identity Law
When the variable is AND with 1 and OR with 0, the variable remains the same, i.e.,
B.1 = B
B+0 = B
B.B = B
B+B = B
4) Complement Law
When the variable is AND and OR with its complement, it will give the result 0 and 1 respectively.
B.B’ = 0
B+B’ = 1
((A)’)’ = A
6) Commutative Law
This law states that no matter in which order we use the variables. It means that the order of variables
does not matter in this law.
A.B = B.A
A+B = B+A
7) Associative Law
This law states that the operation can be performed in any order when the variables priority is of same
as ‘*’ and ‘/’.
(A.B).C = A. (B.C)
(A+B) +C = A+(B+C)
9) Absorption Law
This law allows us for absorbing the similar variables.
B+ (B.A) = B
B.(B+A) = B
(A.B)’ = A’+B’
(A+B)’ = A’. B’
A) Numerical logic
B) Boolean algebra
C) Arithmetic logic
D) Boolean number
A) Logic 0
B) Logic 1
C) Logic X
D) None of the above
A) X’ + Y’
B) X’ – Y’
C) X’ * Y’
D) X’/Y’
A) De Morgan’s theorem
B) Consensus theorem
C) Shannon’s theorem
D) All the above
We defined the Boolean function F=xy’ z+p in terms of four binary variables x, y, z, and p. This func-
tion will be equal to 1 when x=1, y=0, z=1 or z=1.
Example 2:
F (A, B, C, D) = A + BC’ +ACD
Boolean Function = Boolean Expression
Apart from the algebraic expression, the Boolean function can also be described in terms of the truth
table. We can represent a function using multiple algebraic expressions. They are their logical equiv-
alents. But for every function, we have only one unique truth table.
In truth table representation, we represent all the possible combinations of inputs and their result. We
can convert the switching equations into truth tables.
The output will be high when A=1 or BC’=1 or D=1 or all are set to 1. The truth table of the above
example is given below. The 2n is the number of rows in the truth table. The n defines the number of
input variables. So, the possible input combinations are 23=8.
A) OR and XOR
B) NOR and XNOR
C) MAX and MIN
D) SOM and POM
A) 2
B) 5
C) 4
D) 1
A) addition
B) product
C) moduler
D) subtraction
Let us take some examples in which we take some expressions and apply De Morgan’s theorems.
Example 1: (A.B.C)’
(A.B.C)’=A’+B’+C’
Example 2: (A+B+C)’
(A+B+C)’=A’. B’.C
For applying the De Morgan’s theorem on this expression, we must follow the following expressions:
1) In complete expression, first, we find those terms on which we can apply the De Morgan’s theorem
and treat each term as a single variable.
So,
3) Next, we use rule number 9, i.e., (A=(A’)’) for canceling the double bars.
Now, this expression has no term in which we can apply any rule or theorem. So, this is the final ex-
pression.
A) Augustus De Morgan
B) Charles De Morgan
C) Richard De Morgan
D) None of the above
A) Boolean algebra
B) Logic gates
C) Arithmetic
D) Both a and b
12) A NAND gate in De Morgans formula for (X.Y)’ is also called __________.
A) Bubbled OR
B) Bubbled NOR
C) Bubbled XOR
D) Bubbled NOT
13) Which of the following logic gates are required for representing De Morgans NAND gate:
A) AND
B) NOT
C) OR
D) Both a and b
a) Unary falsum
b) Unary identity
c) Unary negation
d) Unary true
Terminal Questions
1. Solve X•Y+X•Y’=X
Self-Assessment Questions
Question No. Answer
1 C
2 A
3 B
4 A
5 D
6 D
7 D
8 A
9 A
10 D
11 D
12 A
13 D
14 A
Activity
1. Use De Morgan’s Theorem, as well as any other applicable rules of Boolean algebra, to
simplify the following expression so there are no more complementation bars extending over
multiple variables:
__________
__ ___
AB + AC
Bibliography
● Mano, M. (1992). Computer System Architecture. PHI.
● William Stallings. (1987). Computer System Architecture. PHI.
● Brown, S., & Vranesic, Z. (2002). Fundamentals of Digital Logic with Verilog Design (2nd
ed). McGraw-Hill.
● Mano, M. (1979). Digital Logic and Computer Design. Pearson.
● Jain, R. P. (2009). Modern Digital Electronics (4th ed). Tata McGraw Hill.
e-References
● https://www.electronics-tutorials.ws/boolean/bool_6.html
● https://www.electronics-tutorials.ws/boolean/bool_7.html
● https://www.electronics-tutorials.ws/boolean/bool_8.html
● https://www.electronics-tutorials.ws/boolean/demorgan.html
Video links
● https://youtu.be/AnQsznjccUw
Image credits
● Fig.1: De Morgan’s First Theorem Logic Circuit
https://www.javatpoint.com/de-morgans-theorem-of-boolean-algebra-in-digital-electronics
https://www.javatpoint.com/de-morgans-theorem-of-boolean-algebra-in-digital-electronics
Module - 2
Unit - 2
Operations of Boolean
Algebra
Instructional Objectives
This unit intends to:
Learning Outcomes
Upon completion of this unit, you will be able to:
1) Boolean Addition
The addition operation of Boolean algebra is similar to the OR operation. In digital circuits, the OR op-
eration is used to calculate the sum term, without using AND operation. A + B, A + B’, A + B + C’, and
A’ + B + + D’ are some of the examples of ‘sum term’. The value of the sum term is true when one or
more than one literal is true and false when all the literals are false.
2) Boolean Multiplication
The multiplication operation of Boolean algebra is like the AND operation. In digital circuits, the AND
operation calculates the product, without using OR operation. AB, AB, ABC, and ABCD are some of the
examples of the product term. The value of the product term is true when all the literals are true and
false when any one of the literals is false.
1) Commutative Law
This law states that no matter in which order we use the variables. It means that the order of variables
does not matter. In Boolean algebra, the OR and the addition operations are similar. In the below dia-
gram, the OR gate display that the order of the input variables does not matter at all.
A+B = B+A
A.B = B.A
2) Associative Law
This law states that the operation can be performed in any order when the variables priority is same.
As ‘*’ and ‘/’ have same priority. In the below diagram, the associative law is applied to the 2-input OR
gate.
A + (B + C) = (A + B) + C
A(BC) = (AB)C
According to this law, no matter in what order the variables are grouped when ANDing more than two
variables. In the below diagram, the associative law is applied to 2-input AND gate.
3) Distributive Law:
According to this law, if we perform the OR operation of two or more variables and then perform the
AND operation of the result with a single variable, then the result will be similar to performing the AND
operation of that single variable with each two or more variable and then perform the OR operation of
that product. This law explains the process of factoring.
A (B + C) = AB + AC
1) Rule 1: A + 0 = A
Let us suppose; we have an input variable A whose value is either 0 or 1. When we perform OR
operation with 0, the result will be the same as the input variable. So, if the variable value is 1, then
the result will be 1, and if the variable value is 0, then the result will be 0. Diagrammatically, this rule
can be defined as:
2) Rule 2: (A + 1) = 1
Let us suppose; we have an input variable A whose value is either 0 or 1. When we perform OR
operation with 1, the result will always be 1. So, if the variable value is either 1 or 0, then the result will
always be 1. Diagrammatically, this rule can be defined as:
Let us suppose; we have an input variable A whose value is either 0 or 1. When we perform the AND
operation with 0, the result will always be 0. This rule states that an input variable ANDed with 0 is
equal to 0 always. Diagrammatically, this rule can be defined as:
4) Rule 4: (A.1) = A
Let us suppose; we have an input variable A whose value is either 0 or 1. When we perform the AND
operation with 1, the result will always be equal to the input variable. This rule states that an input
variable ANDed with 1 is equal to the input variable always. Diagrammatically, this rule can be defined
as:
5) Rule 5: (A + A) = A
Let us suppose; we have an input variable A whose value is either 0 or 1. When we perform the OR
operation with the same variable, the result will always be equal to the input variable. This rule states
an input variable ORed with itself is equal to the input variable always. Diagrammatically, this rule can
be defined as:
Let us suppose; we have an input variable A whose value is either 0 or 1. When we perform the OR
operation with the complement of that variable, the result will always be equal to 1. This rule states that
a variable ORed with its complement is equal to 1 always. Diagrammatically, this rule can be defined
as:
7) Rule 7: (A.A) = A
Let us suppose; we have an input variable A whose value is either 0 or 1. When we perform the AND
operation with the same variable, the result will always be equal to that variable only. This rule states
that a variable ANDed with itself is equal to the input variable always. Diagrammatically, this rule can
be defined as:
8) Rule 8: (A.A’) = 0
Let us suppose; we have an input variable A whose value is either 0 or 1. When we perform the AND
operation with the complement of that variable, the result will always be equal to 0. This rule states
that a variable ANDed with its complement is equal to 0 always. Diagrammatically, this rule can be
defined as:
9) Rule 9: A = (A’)’
This rule states that if we perform the double complement of the variable, the result will be the same
as the original variable. So, when we perform the complement of variable A, then the result will be A’.
Further if we again perform the complement of A’, we will get A, that is the original variable.
We can prove this rule by using the rule 2, rule 4, and the distributive law as:
A) M.(~M+N) = M.N
B) M+ (N.O) = (M+N) (M+O)
C) ~(M+N) = ~M.~N
D) M. (N.O) = (M.N). O
A) OR and NOR
B) AND
C) NAND and NOR
D) NOT
3) Which of the following logic gates are required for representing De Morgans NAND
gate?
A) AND
B) NOT
C) OR
D) Both a and b
4) Which of the following is the output of (X+Y+Z)’ = (X’ Y’ Z’) when (X Y Z) = (0 0 0)?
A) 1
B) 0
C) X
D) None of the above
A) X + Y = Y + X
B) X.Y = Y.X
C) X +(Y+Z) = (X+Y) +Z
D) X.(Y+Z) = (X.Y) + (X.Z)
● Boolean addition operation is similar to the OR operation. In digital circuits, the OR opera-
tion is used to calculate the sum term, without using AND operation.
● Boolean multiplication operation is similar to the AND operation. In digital circuits, the AND
operation is used to calculate the product term, without using OR operation.
● The law of Boolean Algebra includes Associative law, Commutative and Distributive law.
● Commutative law states that the use of variables can be in any order.
● Associative law states that if the variables priority is same, the operation can be performed
in any order.
● To simplify Boolean expressions, there are nine rules that are followed.
Terminal Questions
1. Use Boolean laws to find equivalent expression for XY+YZ+YZ’
3. Draw the logic diagram for the given function F = XY+X’Z+X (Y’+Z’)
Self-Assessment Questions
Question No. Answer
1 A
2 C
3 D
4 A
5 D
Activity
F=ABC+ABC’+AB’C+A’BC+A’BC’
● Associative law: This law states that the operation can be performed in any order when
the variables priority is same.
● Commutative law: If any logical operation of two Boolean variables gives the same result
irrespective of the order of those two variables, then that logical operation is said to be
Commutative.
● Distributive law: If any logical operation can be distributed to all the terms present in the
Boolean function, then that logical operation is said to be Distributive.
● Logic circuit: An integrated circuit which provides a fixed set of output signals according
to the signals present at the input.
Bibliography
e-References
● https://www.electronics-tutorials.ws/boolean/bool_6.html
● https://www.electronics-tutorials.ws/boolean/bool_7.html
● https://www.electronics-tutorials.ws/boolean/bool_8.html
● https://www.electronics-tutorials.ws/boolean/demorgan.html
Image Credits
● Fig.1: Commutative Law (OR)
● Fig.2: Commutative Law (AND)
● Fig.3: Associate Law (AND)
● Fig.4: Associate Law (OR)
● Fig.5: Distributive Law
● Fig.6: Rule-1 A + 0 = A Logic Circuit
● Fig.7: Rule-2 (A + 1) = 1 Logic Circuit
● Fig.8: Rule-3 (A.0) = 0 Logic Circuit
● Fig.9: Rule-4 (A.1) = A Logic Circuit
● Fig.10: Rule-5 (A + A) = A Logic Circuit
● Fig.11: Rule-6 (A + A’) = 1 Logic Circuit
● Fig.12: Rule-7 (A.A) = A Logic Circuit
● Fig.13: Rule-8 (A.A’) = 0 Logic Circuit
● Fig.14: Rule-9 A = (A’)’ Logic Circuit
● Fig.15: Rule-10 (A + AB) = A Logic Circuit
● Fig.16: Rule-11 A + AB = A + B Logic Circuit
● Fig.17: Rule-12 (A + B) (A + C) = A + BC Logic Circuit
● (https://www.javatpoint.com/laws-and-rules-of-boolean-algebra-in-digital-electronics)
Keywords
● Boolean Laws
● De Morgan’s Theorems
● Commutative Law
Module - 2
Unit - 3
Instructional Objectives
This unit intends to:
Learning Outcomes
Upon completion of this unit, you will be able to:
● Summarise the logic gates operation for combinational logic circuits design
● Analyse the Truth table and Logic Design for the several Logic Gates
1) AND Gate
This gate works in the same way as the logical operator “and”. The AND gate is a circuit that performs
the AND operation of the inputs. This gate has a minimum of 2 input values and an output value.
A logic gate used to perform logical multiplication is known as AND gate. An AND gate is a logic circuit
having two or more inputs and one output. The output of an AND gate is HIGH only when all of its
inputs are in the HIGH state. In all other cases, the output is LOW. The logic symbol and truth table
of a two-input AND gate is shown in figure. The AND operation on two independent logic variables A
and B is written as Y = A.B and reads as Y equals A AND B. The operation of a two-input AND gate is
explained by the logic expression.
Y=A.B.C. D……N
Y=ABCD……N
Logic Design
Truth Table
A logic gate used to perform the operation of logical addition is called an OR gate. An OR gate
performs an ORing operation on two or more than two logic variables. The OR operation on two
independent logic variables A and B is written as Y = A+B and reads as Y equals A OR B. An OR
gate is a logic circuit with two or more inputs and one output. The output of an OR gate is LOW only
when all of its inputs are LOW. For all other possible input combinations, the output is HIGH. A truth
table lists all possible combinations of input binary variables and the corresponding outputs of a logic
system. Figure shows the circuit symbol and the truth table of a two-input OR gate. The operation of
a two-input OR gate is explained by the logic expression.
Y=A OR B OR C OR D……N
Y=A+B+C+D……N
Logic Design
Truth Table
A logic gate used to perform logical inversion is known as a NOT gate. A NOT gate is a one -input, one-
output logic circuit whose output is always the complement of the input. That is, a LOW input produces
a HIGH output, and vice versa. If X is the input to a NOT circuit, then its output Y is given by Y = Ā or A’
and reads as Y equals NOT A. The logic symbol and truth table of a NOT gate is shown in figure. The
operation of a NOT gate is explained by the logic expression
Y=NOT A
Y=A’
Logic Design
Truth Table
4) NAND Gate
The NAND gate is the combination of AND gate and NOT gate. This gate gives the same result as a
NOT-AND operation. This gate can have two or more than two input values and only one output value.
NAND Gate is known as Universal gate as it can be used alone to implement any gate operation.
Hence it is said to be functionally complete.
Logic Design
Truth Table
5) NOR Gate
The NOR gate is the combination of an OR gate and NOT gate. This gate gives the same result as the
NOT-OR operation. This gate can have two or more than two input values and only one output value.
NOR gate is also known as Universal gate as it is used alone to implement any gate operation and
hence it is also functionally complete.
NOR stands for NOT OR. An OR gate followed by a NOT circuit makes it a NOR gate. The output of
a NOR gate is logic ‘1’ when all its inputs are logic ‘0’. For all other input combinations, the output is
logic ‘0’. The symbol and truth table of a NOR gate is as shown. The output of a two-input NOR gate
is logically expressed as
Logic Design
Truth Table
6) XOR Gate
The XOR gate is also known as the Ex-OR gate. The XOR gate is used in half and full adder and
subtractor. The exclusive-OR gate is sometimes called as EX-OR and X-OR gate. This gate can have
two or more than two input values and only one output value.
The EXCLUSIVE-OR gate, commonly written as EX-OR gate, is a two-input, one-output gate. The
output of an EX-OR gate is logic ‘1’ when the inputs are unlike and logic ‘0’ when the inputs are
like. Although EX-OR gates are available in integrated circuit form only as two-input gates, unlike
other gates which are available in multiple inputs also, multiple-input EX-OR logic functions can be
implemented using more than one two-input gates. The output of a multiple-input EX-OR logic function
is logic ‘1’ when the number of 1s in the input sequence is odd and logic ‘0’ when the number of 1s in
the input sequence is even, including zero. The symbol and truth table of an EX-OR gate is shown in
figure. The output of a two-input EX-OR gate is logically expressed as
Y=ABCD……N
Y=AB’+A’B
Logic Design
Truth Table
7) XNOR Gate
The XNOR gate is also known as the Ex-NOR gate. The XNOR gate is used in half and full adder and
subtractor. The exclusive-NOR gate is sometimes called as EX-NOR and X-NOR gate. This gate can
have two or more than two input values and only one output value.
The output of a two-input EX-NOR gate is logic ‘1’ when the inputs are like and logic ‘0’ when they are
unlike. In general, the output of a multiple-input EX-NOR logic function is logic ‘0’ when the number of
1s in the input sequence is odd and a logic ‘1’ when the number of 1s in the input sequence is even
including zero.
EXCLUSIVE-NOR (commonly written as EX-NOR) means NOT of EX-OR, i.e., the logic gate that we
get by complementing the output of an EX-OR gate. The truth table of an EX-NOR gate is obtained from
the truth table of an EX-OR gate by complementing the output entries as shown in figure. Logically,
Logic Design
Truth Table
A) One group
B) Two groups
C) Three groups
D) Four groups
A) NOT
B) NAND & NOR
C) X-OR & X-NOR
D) NOT, AND, & OR
A) OR-AND-Invert (OAI)
B) AND-OR-Invert (AOI)
C) Both OAI and AOI
D) None of the above
A) Reliable
B) Not easy to use
C) Not reliable
D) Reliable and easy to use
A) Minterm expansion
B) Disjunctive normal form
C) Both a and b
D) None of the above
A) a+0=0+a=a
B) 1+a=a+1=1
C) ab=ba
D) a+(b+c)=(a+b)+c
A) Compute outputs
B) Compute new states
C) Both a and b
D) None of the above
A) Memory elements
B) Memory is provided by feedback
C) Both a and b
D) None of the above
10) _________ are the methods used to represent negative integer numbers.
A) 1’s compliment
B) Sign magnitude
C) 2’s compliment
D) All of the above
A) Binary
B) Hexadecimal
C) Decimal
D) Octal
A) 0101
B) 1010
C) 0110
D) 1110
A) Binary
B) Hexadecimal
C) Decimal
D) Octal
A) 100 gates
B) 1000 gates
C) 10000 gates
D) More than 100,000 gates
A) 100 gates
B) 10,000 to 100,000 gates
C) 10000 gates
D) None of the above
A) Binary
B) Hexadecimal
C) Decimal
D) Octal
18) The propagation time delay of the silicon gate CMOS is _________.
A) 2ns
B) 7ns
C) 10ns
D) 8ns
A) 74LS11
B) 74LS10
C) 74LS12
D) None of the above
A) 74LS11
B) 74LS10
C) 74LS27
D) None of the above
A) 74LS10
B) 74LS11
C) 74LS27
D) All of the above
A) 74LS20
B) 74LS21
C) 74LS27
D) None of the above
A) 74LS04
B) 74LS19
C) 74LS23
D) None of the above
● Most electronic devices we use today will have some form of logic gates in them.
● The different type of logic gates is AND, OR, NOT, NAND, NOR, XOR, XNOR.
● The logical multiplication operation of the input variables is performed by logical gate
AND.
● The logical addition operation of the input variables is performed by logical gate OR.
● NOR gate is also a universal gate that can perform all the gate operations.
Terminal Questions
Self-Assessment Questions
Question No. Answer
1 C
2 C
3 C
4 A
5 D
6 C
7 B
8 C
9 C
10 D
11 B
12 A
13 A
14 D
15 D
16 B
17 C
18 D
19 A
20 B
21 C
22 A
23 A
Activity
Activity Type: Offline Duration: 40-45 min
1. Construct logic circuit using logic gates for given Boolean equation:
F= AB+C
F=A+BC’
● AND gate: A basic logic operation in which a true (HIGH) output occurs only when al the
input conditions are true (HIGH).
● Exclusive-OR (XOR) gate: A basic logic operation in which a HIGH occurs when the two
inputs are at opposite levels.
● NAND gate: A logic circuit in which a LOW output occurs only if all the inputs are HIGH.
● OR gate: A logic gate that produces a HIGH output when one or more inputs are HIGH.
● NOR gate: A logic gate that produces a HIGH output when all the inputs are LOW.
● NOT gate: A logic gate that produces a HIGH output when the inputs is LOW.
Bibliography
e-References
● https://epgp.inflibnet.ac.in/epgpdata/uploads/epgp_content/S000574EE/P001494/
M015065/ET/1459848930et05.pdf
● https://www.electronics-tutorials.ws/logic/logic_10.html
● http://www.ee.surrey.ac.uk/Projects/CAL/digital-logic/gatesfunc/index.html
● https://youtu.be/0lwhoQ5aQe8
● https://youtu.be/hp0eoiP_v3c
Image Credits
Fig.1: 2-input AND Gate
● https://www.javatpoint.com/and-gate-in-digital-electronics
Fig.2: 2-input OR Gate
● https://www.javatpoint.com/logic-gates-in-digital-electronics
Fig.3: NOT Gate
● https://www.javatpoint.com/not-gate-in-digital-electronics
Fig.4: 2-input NAND Gate
● https://www.javatpoint.com/nand-gate-in-digital-electronics
Fig.5: 2-input NOR Gate
● https://www.javatpoint.com/nor-gate-in-digital-electronics
Fig.6: 2-input XOR Gate
● https://www.javatpoint.com/logic-gates-in-digital-electronics
Fig.7: 2-input XNOR Gate
● https://www.javatpoint.com/logic-gates-in-digital-electronics
Keywords
● Logic Gates
● Truth Tables
● Boolean Equations
● XNOR gate
Module - 3
Unit - 1
The Karnaugh map (KM or K-map) is a method of simplifying Boolean algebra expressions. The
Karnaugh map reduces the need for extensive calculations by taking advantage of humans’ pattern-
recognition capability. It also permits the rapid identification and elimination of potential race conditions.
Race hazards are extremely easy to spot using a Karnaugh map, because a race condition may exist
when moving between any pair of adjacent, but disjoint, regions circumscribed on the map.
Karnaugh maps are used to simplify real-world logic requirements so that they can be implemented
using a minimum number of logic gates. With such wide applications, learning K-Maps gives a better
understanding of the simplification of Boolean functions.
The main criterion in the design of a digital circuit is that its cost should be as low as Possible. For that
the expression used to realise that circuit must be minimal. Since the cost is proportional to number
of gate inputs in the circuit, an expression is considered minimal only if it corresponds to the least
possible number of gate inputs.
Hence, the minimisation techniques are especially useful and helpful to make the circuit minimal in
design and the learner must be aware of all the techniques for developing high-end technology that
makes the emerging technologies like nanotechnology to develop with new inventions in all the fields.
Therefore, this module attempts to introduce the concepts K-Map and the Minimisation techniques that
all the learners must be aware of to gain expertise in this area.
Instructional Objectives
This unit intends to:
● Demonstrate why optimised combinational logic circuits are important in various computer
applications
● Explain K-Map method and types
● Discuss the simplification of Boolean Expressions using K-map
Learning Outcomes
Upon completion of this unit, you will be able to:
Just like the truth table, a K-map contains all the possible values of input variables and their
corresponding output values. However, in K-map, the values are stored in cells of the array. In each
cell, a binary value of each input variable is stored.
The K-map method is used for expressions containing 2, 3, 4, and 5 variables. For a higher number
of variables, there is another method used for simplification called the Quine-McClusky method. In
K-map, the number of cells is similar to the total number of variable input combinations. For example,
if the number of variables is three, the number of cells is 23=8, and if the number of variables is four,
the number of cells is 24. The K-map takes the SOP and POS forms. The K-map grid is filled using
0’s and 1’s. The K-map is solved by making groups. There are the following steps used to solve the
expressions using K-map:
● Let R1, R2, R3 and R4 represents the min terms of first row, second row, third row and fourth
row, respectively. Similarly, C1, C2, C3 and C4 represents the min terms of first column,
second column, third column and fourth column, respectively. The possible combinations
of grouping 8 adjacent min terms are {(R1, R2), (R2, R3), (R3, R4), (R4, R1), (C1, C2), (C2, C3),
(C3, C4), (C4, C1)}.
● There are two possibilities of grouping 16 adjacent min terms. i.e., grouping of min terms
from m0 to m15 and m16 to m31.
In the above all K-maps, we used exclusively the minterms notation. Similarly, you can use exclusively
the maxterms notation.
A) Circuit diagram
B) Block diagram
C) Logic diagram
D) Venn diagram
A) Fourier transform
B) Gray code
C) Karnaugh mapping
D) Venitch method
3) If n denotes the number of variables, then the number of cells is given as:
A) 2n
B) 2 + n
C) 2 – n
D) 2n
A) 12 cells
B) 16 cells
C) 18 cells
D) 20 cells
Step 1:
Step 2:
Next, we create the K-map by entering 1 to each product-term into the K-map cell and fill the remain-
ing cells with zeros.
Step 3:
Notice that each group should have the largest number of ‘ones’. A group cannot contain an empty
cell or cell that contains 0.
We group the number of ones in the decreasing order. First, we have to try to make the group of eight,
then for four, after that two and lastly for 1.
In horizontal or vertical manner, the groups of ones are formed in shape of rectangle and square. We
cannot perform the diagonal grouping in K-map.
The elements located at the edges of the table are adjacent. So, we can group these elements.
We can consider the ‘don’t care condition’ only when they aid in increasing the group-size. Otherwise,
‘don’t care’ elements are discarded.
In the next step, we find the Boolean expression for each group. By looking at the common variables
in cell-labelling, we define the groups in terms of input variables. In the below example, there is a total
of two groups, i.e., group 1 and group 2, with two and one number of ‘ones’.
In the first group, the ones are present in the row for which the value of A is 0. Thus, they contain the
complement of variable A. Remaining two ‘ones’ are present in adjacent columns. In these columns,
only B term in common is the product term corresponding to the group as A’B. Just like group 1, in
group 2, the one’s are present in a row for which the value of A is 1. So, the corresponding variables of
this column are B’C’. The overall product term of this group is AB’C’.
Step 5:
Lastly, we find the Boolean expression for the Output. To find the simplified Boolean expression in the
SOP form, we combine the product-terms of all individual groups. So, the simplified expression of the
above k-map is as follows:
A’+AB’C’
Let us take some examples of 2-variable, 3-variable, 4-variable, and 5-variable K-map examples.
● We will populate the K-map by entering the value of 0 to each sum-term into the K-map cell
and fill the remaining cells with one’s.
● Now, we will define the Boolean expressions for each group as sum-terms.
● At last, to find the simplified Boolean expression in the POS form, we will combine the sum-
terms of all individual groups.
Let us take some examples of 2-variable, 3-variable, 4-variable and 5-variable K-maps.
Simplification Algorithm:
Simplification of logical functions using K-maps is based on the principle of combining terms in adja-
cent cells. Two cells are said to be adjacent if they differ in only one variable.
1. Identify the ones which cannot be combined with any other ones and encircle them. These
are called essential prime implicants.
2. Identify the ones that can be combined in groups of two in only one way. Encircle them.
3. Identify the ones that can be combined with three other ones, to make a group of four ad-
jacent ones, in only one way. Encircle such group of ones.
4. Identify the ones that can be combined with seven other ones, to make a group of eight
adjacent ones, in only one way. Encircle them.
5. After identifying the essential groups of 2, 4, and 8 ones, if there remain some ones which
have not been encircled, then these are to be combined with each other or with other al-
ready encircled ones.
Problem-1
Sol: F=A′C+A′B+AB′C+BC=A′B′C+A′BC′+A′BC+AB′C+AB′C
F=C+A′B
Problem-2
F=A′B′+A′D+ABCD′+B′C′+B′D
Problem-4
F=D+A’C
Problem-6
F=A′B′D′+BCD+BC′D′+B′C′D
5) Do not care conditions can be used for simplifying Boolean expression in:
A) Logic diagram
B) Minterms
C) K-maps
D) Maxterms
A) octet
B) singlet
C) pair
D) quad
7) The Karnaugh map, each cell represents ___________ minterm derived from the Boolean
expression.
A) 1
B) 2
C) 3
D) 4
A) Pairs
B) Triad
C) Quads
D) Octet
9) In a Karnaugh map the formation of Quad results in the elimination of _________ variables
and their complements.
A) 2
B) 3
C) 4
D) 8
10) The binary number designations of the rows and columns of the K-map are in
A) binary code
B) BCD code
C) Gray code
D) XS-3 code.
● K-Map is used for expressions containing 2,3,4 and 5 variables and for higher variables,
Quin-McClusky method is used.
● Solving K-Map is done by making groups, the K-Map grid is filled by 0’s and 1’s.
● Depending on the number of variables, the types of K- Map are two variable K-Map, three
variable K-Map, four variable K-Map, five variable K-Map.
● Minterm solutions and Maxterm solutions of K-Map are the simplification methods of
Boolean expression using K-Map.
● The principle of combining terms in adjacent cells is used as a method of logical functions
to simplify K-Maps.
Terminal Questions
1. Using K-map, minimise the following function F (A, B, C) = ∏ M (0,1,3,6)
Self-Assessment Questions
Question No. Answer
1 D
2 C
3 A
4 B
5 C
6 D
7 A
8 D
9 A
10 C
Activity
Using K-MAP method Construct an Optimised logic circuit for a given SOP and POS:
● SOP: Sum of Products, one of the forms of Boolean expression consisting purely of
Minterms (product terms).
● POS: Product of Sums, one of the forms of Boolean expression consisting purely of
Maxterms (sum terms).
Bibliography
e-References
● https://www.usna.edu/Users/cs/lmcdowel/courses/ic220/S21/resources/kmaps.html
● https://www.allaboutcircuits.com/technical-articles/karnaugh-map-boolean-algebraic-
simplification-technique/
Video Links
● https://youtu.be/y-aYzGdlM-8
● https://youtu.be/wjM2RDG5yTI
● https://youtu.be/JRR8RCKMKjA
● https://www.javatpoint.com/simplification-of-boolean-expressions-using-karnaugh-map
● https://www.javatpoint.com/simplification-of-boolean-expressions-using-karnaugh-map
● https://www.javatpoint.com/simplification-of-boolean-expressions-using-karnaugh-map
● https://www.javatpoint.com/simplification-of-boolean-expressions-using-karnaugh-map
● https://www.javatpoint.com/simplification-of-boolean-expressions-using-karnaugh-map
● https://www.javatpoint.com/simplification-of-boolean-expressions-using-karnaugh-map
● https://www.javatpoint.com/simplification-of-boolean-expressions-using-karnaugh-map
● https://www.javatpoint.com/simplification-of-boolean-expressions-using-karnaugh-map
● https://www.javatpoint.com/simplification-of-boolean-expressions-using-karnaugh-map
Keywords
● K-Maps
● Boolean Equations
● Maxterms
● Minterms
● Combinational logic
Module - 3
Unit - 2
Minimisation Techniques
3.2.1 POS and SOP form representation of a Boolean Function -------------------- 164
Self-Assessment Questions ----------------------------------------------------------- 168
3.2.2 Min and Max Terms Notation in Boolean algebra -------------------------------- 169
Self-Assessment Questions ----------------------------------------------------------- 176
Instructional Objectives
This unit intends to:
Learning Outcomes
Upon completion of this unit, you will be able to:
As the name suggests, A POS expression contains the sum of various terms ANDed/multiplied
together.
As the name suggests, A SOP expression is a group of product terms ORed/added together.
If each term of SOP/POS expression contains all the literals in the Boolean function, then they
are said to be in canonical form. Suppose we have a Boolean function (Y) having three literals A, B
and C, then canonical Boolean expressions can be written as,
Y = ABC + A. B. C + A.B.C
This is an example of canonical SOP Form, because each term in the SOP form contains all the lit-
erals A, B, and C.
This is an example of canonical POS form, because each term of the POS form contains all the lit-
erals.
1. Y (A, B, C) = AB + BC + CA
2. Y (X, Y, Z) = X. (X + Y). (X + Y + Z)
Y (A, B, C) = AB + BC + CA, this expression is a SOP expression, since we notice the Boolean
function has three literals A, B and C, so each term of the Boolean expression must contain all the
three literals to convert it into canonical SOP form. Therefore,
= Y (A, B, C) = AB + BC + CA
= ABC + AB. C + A. BC + A. B. C
Hence,
Solution (ii):
Y (X, Y, Z) = X. (X + Y). (X + Y + Z), is an example of POS expression, since all the sum terms in
the expression does not have all the literals X, Y, and Z, so we must express it in such a way that it
will have all the three literals in each term.
= Y (X, Y, Z) = X. (X + Y). (X + Y + Z)
= (X + Y. Y + Z. Z) (X + Y + Z. Z). (X + Y + Z)
= (X + Y. Y + Z) (X + Y. Y + Z) (X + Y + Z) (X + Y + Z) (X + Y + Z)
= (X + Y +Z) (X + Y + Z) (X + Y + Z) (X + Y + Z) (X + Y + Z) (X + Y + Z)
= (X + Y +Z) (X + Y + Z) (X + Y + Z) (X + Y + Z)
Hence,
Y = (X + Y +Z) (X + Y + Z) (X + Y + Z) (X + Y + Z)
For Example:
X (SOP) = ∑m (1, 3, 6)
X = A’. B.’C + A.’B.C + A.B.C’
For Example:
A) A(B+C) +CĀ
B) (A+B) (Ā+B+C)
C) Ā+B+BC
D) Both (a) and (b)
A) A+B(C+D)
B) A’B+AC’+AB’C
C) (A’+B+C) (A+B’+C)
D) Both (a) and (b)
4) Using the transformation method you can realise any POS realisation of OR-AND with only:
A) XOR
B) NAND
C) AND
D) NOR
A) AND-NOT realisation
B) AND-OR realisation
C) OR-AND realisation
D) NOT-OR realisation
A) NAND-NOR realisation
B) NAND-NAND realisation
C) NOR-NAND realisation
D) NOR-NOR realisation
7) The AND-OR realisation of a combinational circuit requires three 3-input AND gates and
one 3-input OR gate. This circuit can be designed using:
Minterm
Each of the product terms in the canonical SOP form is called a minterm. Minterm are represented
as binary numbers in terms of 0s and 1s. The binary words are formed by representing each non-
complemented variable by 1 and each complemented variable by 0, and the decimal equivalent of
this binary word is represented as a subscript of m as m0, m1, m2, etc. We use the ∑ (sigma) notation
to represent minterms.
Maxterm
Each of the sum terms in the canonical POS form is called a maxterm. Maxterm can also be
represented using binary numbers where each non-complemented variable is represented using 0 and
complemented variable using 1, and the decimal equivalent of this binary word is represented as a
subscript of M as M0, M2, M2, etc. We use ∏ (pi) notation to represent the max terms.
Note: For n-variable logic function there are 2n minterms and 2n maxterms.
Table 2: Min and Max terms for two literal binary expressions
Solution (a):
Y = ABC + A. B.C + A.B. C + A. B. C, is an example of canonical SOP expression, so its each term can
be represented in minterm notation. Therefore,
= m7 + m3 + m5 + m4
Solution (b):
Y= (A+B+C) (A+ B+ C) (A+ B+ C), is an example of canonical POS expression, so its each term can
be represented in maxterm notation.
= M0 + M5 + M7
Canonical Form – In Boolean algebra, Boolean function can be expressed as Canonical Disjunctive
Normal Form known as minterm and some are expressed as Canonical Conjunctive Normal Form
known as maxterm.
In Minterm, we look for the functions where the output results in “1” while in Maxterm we look for
function where the output results in “0”.
Boolean functions expressed as a sum of minterms or product of maxterms are said to be in canonical
form.
Standard Form – A Boolean variable can be expressed in either true form or complemented form. In
standard form Boolean function will contain all the variables in either true form or complemented form
while in canonical number of variables depends on the output of SOP or POS.
A Boolean function can be expressed algebraically from a given truth table by forming a:
● minterm for each combination of the variables that produces a 1 in the function and then
taking the OR of all those terms.
● maxterm for each combination of the variables that produces a 0 in the function and then
taking the AND of all those terms.
From the above table it is clear that minterm is expressed in product format and maxterm is expressed
in sum format.
Sum of minterms
The minterms whose sum defines the Boolean function are those which give the 1’s of the function
in a truth table. Since the function can be either 1 or 0 for each minterm, and since there are 2^n
minterms, one can calculate all the functions that can be formed with n variables to be (2^(2^n)). It is
sometimes convenient to express a Boolean function in its sum of minterm form.
Solution –
A = A(B + B’) = AB + AB’
This function is still missing one variable, so
A = AB(C + C’) + AB’(C + C’) = ABC + ABC’+ AB’C + AB’C’
The second term B’C is missing one variable; hence,
B’C = B’C(A + A’) = AB’C + A’B’C
Combining all terms, we have
F = A + B’C = ABC + ABC’ + AB’C + AB’C’ + AB’C + A’B’C
But AB’C appears twice, and
according to theorem (x + x = x), it is possible to remove one of those occurrences. Rearranging the
minterms in ascending order, we finally obtain
F = A’B’C + AB’C’ + AB’C + ABC’ + ABC
= m1 + m4 + m5 + m6 + m7
SOP is represented as ∑m (1, 4, 5, 6, 7)
Solution –
F = xy + x’z = (xy + x’)(xy + z) = (x + x’)(y + x’)(x + z)(y + z) = (x’ + y)(x + z)(y + z)
x’ + y = x’ + y + zz’ = (x’+ y + z)(x’ + y + z’)
x + z = x + z + yy’ = (x + y + z)(x + y’ + z)
y + z = y + z + xx’ = (x + y + z)(x’ + y + z)
F = (x + y + z)(x + y’ + z)(x’ + y + z)(x’ + y + z’)
= M0*M2*M4*M5
POS is represented as ∏M (0, 2, 4, 5)
Example –
Solution: The function has three variables: A, B, and C. The first term A is missing two variables;
therefore,
A = A (B + B’) = AB + AB’
But AB’C appears twice, and according to theorem (x + x = x), it is possible to remove one of those
occurrences. Rearranging the minterms in ascending order, we finally obtain
= m1 + m4 + m5 + m6 + m7
Solution: First, convert the function into OR terms by using the distributive law:
The function has three variables: x, y, and z. Each OR term is missing one variable; therefore,
x’+ y = x’ + y + zz’
= x + z + yy’ = (x + y + z) (x + y’ + z) y + z
Combining all the terms and removing those which appear more than once, we finally obtain
F= M0.M2.M4.M5 A
The product symbol, π, denotes the ANDing of maxterms; the numbers are the indices of the maxterms
of the function.
F = xy’ + yz’
= Σm (2, 4, 5, 6)
= πM (0, 1, 3, 7)
In canonical form each sum term is a max term so it can also be written as:
F = ∏M (0,4,6,7)
The remaining combinations of inputs are minterms of the function for which its output is true. To
convert it into SOP expression first we will change the symbol to summation (∑) and use the remaining
minterm.
F = ∑m (1,2,3,5)
Now we will expand the summation sign to form canonical SOP expression.
8) Each product term of a group, w’.x.y’ and w.y, represents the _________ in that group.
A) Input
B) POS
C) Sum-of-Minterms
D) Sum of Maxterms
A) Minterm
B) Maxterm
C) Don’t care.
D) Literal
A) Minterm
B) Maxterm
C) Don’t care.
D) Literal
Terminal Questions
Answer Keys
Self-Assessment Questions
Question No. Answer
1 B
2 B
3 D
4 D
5 D
6 B
7 D
8 C
9 A
10 B
F=AB+CD
H=(A+B). (C+D)
Glossary
● Included minterm: one of the terms in a canonical SOP is the Included maxterm
● Excluded minterm: A minterm not in a particular canonical SOP is the Excluded maxterm
● Canonical SOP: a sum of products, the terms of which are all minterms, that is, each term
contains all input variables with or without the complement operator.
● Canonical POS: a product of sums, the factors of which are all maxterms, that is, each
term includes all input variables OR’d together with or without the complement operator
on each variable.
Bibliography
● Mano, M. (1992). Computer System Architecture. PHI.
● Stallings, W. (2015). Computer System Architecture. PHI.
● Brown, S., & Vranesic, Z. (2008). Fundamentals of Digital Logic with Verilog Design (2nd
ed.). McGraw-Hill.
● Mano, M. (2017). Digital Logic and Computer Design. Pearson.
● Jain, R. P. (2009). Modern Digital Electronics (4th ed.). Tata McGraw Hill.
Video links
● https://youtu.be/K2cpJex0o_A
● https://youtu.be/YmKmS9bpMqM
Keywords
● SOP
● POS
● Boolean variable
● Sum of minterm
● Product of maxterm
● NAND realisation
Module - 4
Unit - 1
Combinational Circuits
&
Sequential Circuits
A digital circuit is designed using several logic gates on a single integrated circuit – IC. The input to
any digital circuit is in the binary form “0’s” and “1’s”. The output obtained from processing raw digital
data is of precise value. These circuits can be represented in 2 ways either in a combinational way or
a sequential way.
Combinational and Sequential circuits are the most used type of digital circuits in digital electronics.
The sequential circuits and combinational circuits are being used in the displays such as boarding
flights, bus stations, in weight machines, in weather forecast instruments, etc.
With real life application like a traffic light controller, a vending machine (the money acceptance and
change return), a password checker (in an electronic door lock) etc., the growing importance and
usage of these type of digital circuits acquires prominence without any doubt.
The development of the digital electronics on large scale promotes the Research and Development
work in other areas like Health, Education, Space etc.; that paves way for new inventions in the
respective fields that would serve the global needs. Therefore, it is vital to learn each, and every
development related to the digital circuits with more focus on the combinational and sequential circuits
to make oneself more knowledgeable in the relevant domain.
Hence, this module attempts to introduce the basic concepts of digital circuits with emphasis on the
combinational and sequential circuits.
Instructional Objectives
This unit intends to:
● Explain the architectural and logic design of combinational logic circuit - Adder.
● Describe their relevance and applications in new age.
Learning Outcomes
Upon completion of this unit, you will be able to:
● Illustrate architectural and logic design of combinational logic circuit - Adder.
● Summarise their relevance and applications in new age.
The result is that combinational logic circuits have no feedback, and any changes to the signals being
applied to their inputs will immediately have an effect at the output. In other words, in a Combinational
Logic Circuit, the output is always dependant on the combination of its inputs. Thus, a combinational
circuit is memoryless.
So, if one of its inputs condition changes states, from 0-1 or 1-0, so too will the resulting output as by
default combinational logic circuits have “no memory”, “timing” or “feedback loops” within their design.
Combinational Logic Circuits are made up from basic logic NAND, NOR or NOT gates that are “com-
bined” or connected to produce more complicated switching circuits. These logic gates are the build-
ing blocks of combinational logic circuits. An example of a combinational circuit is a decoder, which
converts the binary code data present at its input into several different output lines, one at a time
producing an equivalent decimal code at its output.
Combinational logic circuits can be quite simple or complicated and any combinational circuit can be
implemented with only NAND and NOR gates as these are classed as “universal” gates.
The three main ways of specifying the function of a combinational logic circuit are:
1) Boolean Algebra – This forms the algebraic expression showing the operation of the logic circuit
for each input variable either True or False those results in a logic “1” output.
Example:
Y=A+B
2) Truth Table – A truth table defines the function of a logic gate by providing a concise list that shows
all the output states in tabular form for each possible combination of input variable that the gate could
encounter.
3) Logic Diagram – This is a graphical representation of a logic circuit that shows the wiring and
connections of each individual logic gate, represented by a specific graphical symbol, that implements
the logic circuit.
Example:
As combinational logic circuits are made up from individual logic gates only, they can also be consid-
ered as “decision making circuits” and combinational logic is about combining logic gates together to
process two or more signals to produce at least one output signal according to the logical function of
each logic gate. Common combinational circuits made up from individual logic gates that carry out a
desired application include Half and Full Adders, Subtractors, Multiplexers, De-multiplexers, Encod-
ers, and Decoders etc.
A half adder is a digital logic circuit that performs binary addition of two single-bit binary numbers. It
has two inputs, A and B, and two outputs, SUM and CARRY. The SUM output is the least significant
bit (LSB) of the result, while the CARRY output is the most significant bit (MSB) of the result, indicating
whether there was a carry-over from the addition of the two inputs. The half adder can be implemented
using basic gates such as XOR and AND gates.
Half adder is the simplest of all adder circuits. Half adder is a combinational arithmetic circuit that adds
two numbers and produces a sum bit (s) and carry bit (c) both as output. The addition of 2 bits is done
using a combination circuit called a Half adder. The input variables are augend and addend bits, and
output variables are sum & carry bits. A and B are the two input bits.
Let us consider two input bits A and B, then sum bit (s) is the X-OR of A and B. it is evident from the
function of a half adder that it requires one X-OR gate and one AND gate for its construction.
Truth Table:
Here we perform two operations Sum and Carry, thus we need two K-maps one for each to derive the
expression.
Logical Expression:
Implementation:
Note: Half adder has only two inputs and there is no provision to add a carry coming from the lower
order bits when multi addition is performed.
● Half adder is used in ALU (Arithmetic Logic Unit) of computer processors to add binary bits.
Full Adder:
Introduction
Addition is one of the most basic operations performed by different electronic devices like computers,
calculators, etc. The electronic circuit that performs the addition of two or more numbers, more
specifically binary numbers, is called as adder. As we know, the logic circuits use binary number sys-
tem to perform the operations, hence the adder is referred to as binary adder.
In other words, a combinational circuit which is designed to add three binary digits and produces two
outputs (sum and carry) is known as a full adder. Thus, a full adder circuit adds three binary digits,
where two are the inputs and one is the carry forwarded from the previous addition.
Full Adder Logic Design: Full adder takes three inputs namely A, B, and Cin. Where, A and B are the
two binary digits, and Cin is the carry bit from the previous stage of binary addition. The sum output of
the full adder is obtained by XORing the bits A, B, and Cin. While the carry output bit (Cout) is obtained
using AND and OR operations.
Truth Table: Truth table is one that indicates the relationship between input and output variables of
a logic circuit and explains the operation of the logic circuit. The following is the truth table of the full-
adder circuit:
Hence, from the truth table, the sum output of the full adder is equal to 1 when only 1 input is equal
to 1 or when all the inputs are equal to 1. While the carry output has a carry of 1 if two or three inputs
are equal to 1.
The characteristic equations of the full adder, i.e., equations of sum (S) and carry output (Cout) are
obtained according to the rules of binary addition. These equations are given below:
The sum (S) of the full-adder is the XOR of A, B, and Cin. Therefore,
Sum, S = A⊕B⊕Cin=A’B’Cin+A’BC’in+ABCin
Carry, C=AB+ACin+BCin
Implementation:
● Full adder provides facility to add the carry from the previous stage.
● The power consumed by the full adder is less as compared to half adder.
● Full adder can be easily converted into a half subtractor just by adding a NOT gate in the
circuit.
● Full adder is one of the essential parts of critic digital circuits like multiplexers.
● Full adders are used in ALUs (arithmetic logic units) of CPUs of computers.
● Full adders are also used to realize critic digital circuits like multiplexers.
A) Addresses
B) Table indices
C) Increment and decrement operators
D) All of the Mentioned
A) Subtraction
B) Addition
C) Multiplication
D) Both addition and subtraction
A) Overflow
B) Output
C) Zero bit
D) Carry
7) If A and B are the inputs of a half adder, the sum is given by __________.
A) A AND B
B) A OR B
C) A XOR B
D) A EX-NOR B
8) If A and B are the inputs of a half adder, the carry is given by __________.
A) A AND B
B) A OR B
C) A XOR B
D) A EX-NOR B
A) Inputs
B) Outputs
C) Both a and b
D) Digits
A) XOR gate
B) XNOR gate
C) NAND gate
D) NOT gate
A) Half adder has two inputs while full adder has four inputs
B) Half adder has one output while full adder has two outputs
C) Half adder has two inputs while full adder has three inputs
D) All of the Mentioned
12) If A, B and C are the inputs of a full adder then the sum is given by __________.
A) A AND B AND C
B) A OR B AND C
C) A XOR B XOR C
D) A OR B OR C
13) If A, B and C are the inputs of a full adder then the carry is given by __________.
A) A AND B OR (A OR B) AND C
B) A OR B OR (A AND B) C
C) (A AND B) OR (A AND B) C
D) A XOR B XOR (A XOR B) AND C
14) How many AND, OR and EXOR gates are required for the configuration of full adder.
A) 1, 2, 2
B) 2, 1, 2
C) 3, 1, 2
D) 4, 0, 1
A) Two half-adders
B) Two half-adders and a NOT gate
C) Two half-adders and an OR gate
D) Two half-adders and an AND gate
17) A logic circuit that provides a HIGH output for both inputs HIGH or both inputs LOW is
____________.
A) Ex-NOR gate
B) OR gate
C) Ex-OR gate
D) NAND gate
20) What is the major difference between half-adders and full adders?
● Logic gates NAND, NOR or NOT gates are combined to form combinational logic circuits.
● Boolean Algebra, Truth Table and Logic Diagram are used to specify combinational logic
circuit.
● A half adder is a type of adder, an electronic circuit that performs the addition of numbers.
● The half adder can add two single binary digits and provide the output plus a carry value.
It has two inputs, called A and B, and two outputs S (sum) and C (carry).
● Two K-Maps are required to derive the logical expression of Sum and Carry.
● The rules of binary addition are used to derive the characteristic equation of Half adder.
● The applications of Half adder include calculators, full adder, ALU of the computer.
● A full adder takes two binary numbers plus a carry or overflow bit. The output is a sum and
another carry bit.
● Full adders are made from XOR, AND and OR gates in hardware.
● Full adders are commonly connected to each other to add bits to an arbitrary length of bits,
such as 32 or 64 bits.
Terminal Questions
1. Draw half adder logic circuit.
2. Explain truth table of half adder.
3. Write Boolean equation of half adder sum and carry.
4. Draw full adder logic circuit.
5. Explain truth table of full adder.
6. Write Boolean equation of full adder sum and carry.
7. List the full adder advantage and applications.
Self-Assessment Questions
Question No. Answer
1 B
2 D
3 C
4 C
5 D
6 D
7 C
8 A
9 C
10 A
11 C
12 C
13 A
14 B
15 C
16 C
17 A
18 B
19 D
20 C
Activity
Activity type: Online/offline Duration: 60 mins
Bibliography
● Mano, M. (1992). Computer System Architecture. PHI.
● Stallings, W. (2015). Computer System Architecture. PHI.
● Brown, S., & Vranesic, Z. (2008). Fundamentals of Digital Logic with Verilog Design (2nd
ed.). McGraw-Hill.
● Mano, M. (2017). Digital Logic and Computer Design. Pearson.
● Jain, R. P. (2009). Modern Digital Electronics (4th ed.). Tata McGraw Hill.
e-References
● https://www.elprocus.com/introduction-to-combinational-logic-circuits/
● https://www.elprocus.com/half-adder-and-full-adder/
Image credits
● Fig.1: Block Diagram of Combinational Logic Circuit
● https://www.electronics-tutorials.ws/combination/comb_1.html
● Fig.2: Logic diagram of OR Gate
● https://cse14-iiith.vlabs.ac.in/exp/transistor-level-xor/
● Fig.3: Logical expression of Half Adder.
● https://silo.tips/download/tema-11-sistemas-combinacionales
● Fig.4: Block diagram and Circuit diagram of half adder
● https://sites.ualberta.ca/~gingrich/courses/phys395/notes/node129.html
Keywords
● Half Adder
● Full Adder
● Truth Table
● Boolean Equation
● Logic Circuit
Module - 4
Unit - 2
Combinational Circuits
Subtractors
Instructional Objectives
This unit intends to:
● Explain the architectural and logic design of combinational logic circuit - Subtractor
● Describe their relevance and applications in new age
Learning Outcomes
Upon completion of this unit, you will be able to:
● Illustrate architectural and logic design of combinational logic circuit - Subtractor
● Analyse their relevance and applications in new age
Truth Table:
Now, let us understand the operation of the half subtractor circuit. Half subtractor performs its operation
to find the difference of two binary digits according to the rules of binary subtraction, which are as
follows:
The output borrow of b is zero (0) if the minuend bit (A) is greater than or equal to the subtrahend bit
(B), i.e. A ≥ B. The output borrow is a 1 when A = 0 and B = 1.
From the logic circuit diagram of the half subtractor, the difference bit (d) is obtained by the XOR
operation of the two inputs A and B, and the borrow bit is obtained by AND operation of the compliment
of the minuend (A’) with the subtrahend (B).
The characteristic equations of the half subtractor, i.e., equations of the difference bit (d) and the
output borrow bit (b) are obtained by following the rules of binary subtraction. These equations are
given as follows:
The difference bit (d) of the half subtractor is given by XORing the two inputs A and B.
Therefore,
The borrow (b) of the half subtractor is the AND of A’ (compliment of A) and B. Therefore,
Barrow, b = A’B
Implementation:
Hence, from the logic circuit diagram, a half subtractor can be realized using an XOR gate together
with a NOT gate and an AND gate.
In the half subtractor as shown in the figure, A and B are the inputs, d and b are the outputs. Where,
d indicates the difference and b indicates the borrow output. The borrow output (b) is the signal that
tells the next stage that a 1 has been borrowed.
● Half subtractor can also be used in amplifiers to compensate the sound distortion.
Full Subtractor
Introduction:
A full-subtractor is a combinational circuit that has three inputs A, B, and bin and two outputs d and
b, where, A is the minuend, B is subtrahend, bin is borrow produced by the previous stage, d is the
difference output and b are the borrow output.
Since, the half subtractor can only be used to find the difference of LSBs (Least Significant Bits) of
two binary numbers. Thus, if there is any borrow during the subtraction of the LSBs, it will affect the
subtraction of the next bits of numbers. To overcome this problem of the half subtractor, a full subtractor
is realised.
From the logic circuit diagram of the full subtractor, the difference bit (d) is obtained by the XOR
operation of the two inputs A, B, and bin, and the output borrow bit (b) is obtained by NOT, AND, and
OR operations of variable A, B, and bin.
Truth Table:
The truth table is one that gives the relationship between the input and output of a logic circuit. The
following is the truth table of the full-subtractor:
Logical Expression:
Implementation:
We can realise the full-subtractor using two XOR gates, two NOT gates, two AND gates, and one OR
gate.
● Full subtractors are used in ALU (Arithmetic Logic Unit) in computers CPUs (Central
Processing Unit).
● Full subtractors are extensively used to perform arithmetical operations like subtraction in
electronic calculators and many other digital devices.
● Full subtractors are also used in processors to compute addresses, tables, etc.
• Full subtractors are also used in DSP (Digital Signal Processing) and networking-based
systems.
A) Difference = 0, borrow = 0
B) Difference = 1, borrow = 0
C) Difference = 1, borrow = 1
D) Difference = 0, borrow = 1
A) 1
B) 4
C) 3
D) 2
A) The minuend and the subtrahend are both changed to the 2’s-complement
B) The minuend is changed to 2’s-complement and the subtrahend is left in its
original form
C) The minuend is left in its original form and the subtrahend is changed to its
2’s-complement
D) The minuend and subtrahend are both left in their original form
A) 2 bits
B) 3 bits
C) 4 bits
D) 5 bits
A) Carry
B) Borrow
C) Input
D) Output
A) 1
B) 2
C) 3
D) 4
A) A XOR B
B) A AND B
C) A OR B
D) A EXNOR B
8) Let A and B is the input of a subtractor then the borrow will be ___________.
A) A AND B’
B) A’ AND B
C) A OR B
D) A AND B
10) Which condition do Bout (borrow output) is set to “1”, when X and Y are 2 inputs?
A) x
B) x>y
C) x=y
D) x=0
A) 2 bits
B) 3 bits
C) 4 bits
D) 8 bits
13) The output of a subtractor is given by (if A, B and X are the inputs).
A) A AND B XOR X
B) A NOR B XOR X
C) A OR B NOR X
D) A XOR B XOR X
A) Half adder
B) Half subtractor
C) Full adder
D) Decoder
15)When the inputs A, B, Bin is (0,0,0) then the Difference output of full subtractor
is________?
A) 0
B) 1
C) x
D) All the above
16) When the inputs A, B, Bin is (0,1,0) then the Borrow output of full subtractor is________?
A) 0
B) x
C) 1
D) All the above
17) Which of the following is used to represent full subtractor logical circuits equation?
A) K-map
B) Q-map
C) Graph
D) All the above
18) Which of the following gates do full subtractor Difference component requires to obtain
output?
A) XOR
B) AND
C) NOT
D) OR
19) Which of the following is the first step of Borrow equation of full subtractor?
A) Binary
B) Analog
C) Signal
D) Both a and b
Terminal Questions
1. Draw half subtractor logic circuit.
2. Explain truth table of half subtractor.
3. Write Boolean equation of half subtractor difference and barrow.
4. Draw full subtractor logic circuit.
5. Explain truth table of full subtractor.
6. Write Boolean equation of full subtractor difference and barrow.
7. List the full subtractor advantage and applications.
Self-Assessment Questions
Question No. Answer
1 A
2 B
3 C
4 A
5 B
6 B
7 A
8 B
9 C
10 A
11 B
12 B
13 D
14 C
15 A
16 C
17 A
18 A
19 A
20 D
Activity
Activity type: Online/offline Duration: 50 mins
Bibliography
● Mano, M. (1992). Computer System Architecture. PHI.
● Stallings, W. (2015). Computer System Architecture. PHI.
● Brown, S., & Vranesic, Z. (2008). Fundamentals of Digital Logic with Verilog Design (2nd
ed.). McGraw-Hill.
● Mano, M. (2017). Digital Logic and Computer Design. Pearson.
● Jain, R. P. (2009). Modern Digital Electronics (4th ed.). Tata McGraw Hill.
e-References
● https://www.elprocus.com/half-subtractor-circuit-construction-using-logic-gates/
Video links
● https://youtu.be/SV4VTYWxKV4
● https://youtu.be/dBXGGWbtt6U
● https://teachics.org/computer-organization-and-architecture-tutorial/half-subtractor/
● https://geek-docs.com/computer/digital-electronic/t_realization-of-a-full-subtractor-us-
ing-two-half-subtractors.html
● https://www.eeeguide.com/half-subtractor-and-full-subtractor-circuit/
● https://geek-docs.com/computer/digital-electronic/t_realization-of-a-full-subtractor-us-
ing-two-half-subtractors.html
Keywords
● Half Subtractor
● Full Subtractor
● Truth Table
● Boolean Equation
● Logic Circuit
Module - 4
Unit - 3
Combinational Circuits
Multiplexers and
De-multiplexers
Instructional Objectives
This unit intends to:
● Explain the architectural and logic design of combinational logic circuit – Multiplexer and
De-multiplexer
Learning Outcomes
Upon completion of this unit, you will be able to:
● Illustrate architectural and logic design of combinational logic circuit – Multiplexer and
De-multiplexer
Unlike encoder and decoder, there are n selection lines and 2n input lines. So, there is a total of 2N
combinations of inputs. A multiplexer is also treated as Mux.
In 2×1 multiplexer, there are only two inputs, i.e., A0 and A1, 1 selection line, i.e., S0 and single
outputs, i.e., Y. Based on the combination of inputs which are present at the selection line S0, one
of these 2 inputs will be connected to the output. The block diagram and the truth table of the 2×1
multiplexer is given below.
Logical Expression:
The logical expression of the term Y is as follows:
Implementation:
Y = x.’1 + x.0 = x’
The implementation of NOT gate is done using “n” selection lines. It cannot be implemented using “n-1”
selection lines. Only NOT gate cannot be implemented using “n-1” selection lines.
Implementation of NAND, NOR, XOR and XNOR gates requires two 2:1 Mux. First, the multiplexer will
act as a NOT gate which will provide complemented input to the second multiplexer.
Fig. 13: Block diagram of SOP Logic using 4x1 multiplexer [AB as Select]
Fig. 14: Block diagram of SOP Logic using 4x1 multiplexer [AC as Select]
Fig. 15: Block diagram of SOP Logic using 4x1 multiplexer [BC as Select]
Introduction:
A De-multiplexer is a combinational circuit that has only 1 input line and 2N output lines. Simply, the
multiplexer is a single-input and multi-output combinational circuit. The information is received from
the single input lines and directed to the output line. Based on the values of the selection lines, the
input will be connected to one of these outputs. De-multiplexer is opposite to the multiplexer.
Unlike encoder and decoder, there are n selection lines and 2n outputs. So, there is a total of 2n
combinations of inputs. De-multiplexer is also treated as De-mux.
Logical Expression
The logical expression of the term Y is as follows:
Y0= E.S0’. A
Y1= E.S0.A
Implementation:
Y0=S1’ S0’ A
y1=S1’ S0 A
y2=S1 S0’ A
y3=S1 S0 A
A) One digital information from several sources and transmits the selected one
B) Many digital information and convert them into one
C) Many decimal inputs and transmits the selected information
D) Many decimal outputs and accepts the selected information
A) Data controller
B) Selected lines
C) Logic gates
D) Both data controller and selected lines
3) If the number of n selected input lines is equal to 2^m then it requires _________ select
lines.
A) 2
B) m
C) n
D) 2n
A) 2
B) 4
C) 8
D) 3
A) Single-pole relay
B) DPDT switch
C) Rotary switch
D) Linear stepper
A) 3
B) 4
C) 2
D) 5
A) Strobe
B) Decoded input
C) Select input
D) Sink
8) If inputs (a,b) are (0,1) for a 2by 1 mux with selector ‘0’. Then the output is __________.
A) 1
B) 0
C) x
D) None of the above
A) Active
B) Passive
C) Both a and b
D) Zero components
A) Decimal-to-hexadecimal
B) Single input, multiple outputs
C) AC to DC
D) Odd parity to even parity
A) 2
B) 3
C) 4
D) 5
A) Data inputs
B) Select inputs
C) Select outputs
D) Enable pin
A) Receiver
B) Transmitter
C) Both a and b
D) None of the above
17) When the enable input is zero, s1=0, S0=0 in the 1:4 demultiplexer then the outputs Y0,
Y1, Y2, and Y3 are ________________?
A) Y0=Y1=Y2=Y3=1
B) Y0=Y1=Y2=Y3=0
C) Y1=Y2=Y3=0, Y0=1
D) Y1=Y2=Y3=1, Y0=0
A) Used as decoders
B) Increases the communication system efficiency
C) Wastage of bandwidth may occur
D) All of the above
19) Which one of the following logics circuits the memory element is not present?
Summary
● A multiplexer makes it possible for several input signals to share one device or resource,
for example, one analogue-to-digital converter or one communications transmission medi-
um, instead of having one device per input signal.
● Multiplexers can also be used to implement Boolean functions of multiple variables.
● In the 2×1 multiplexer, two inputs, one selection lines and one single output constitutes the
operation.
● In the 4×1 multiplexer, four inputs, two selection lines and one single output constitutes the
operation.
● As the name infers, a demultiplexer performs the opposite function to that of a multiplexer.
A single data line can be connected to any one of the output lines provided by the choice
of an appropriate select signal. If there are s select inputs, then the number of output lines
to which the data can be routed is n = 2s.
● 1 to 2 De-multiplexers comprise one single input, two outputs and one selection line.
● 1 to 4 De-multiplexers comprise one single input, four outputs and two selection lines.
Terminal Questions
1. Draw a 2x1 multiplexer logic circuit with enable.
Self-Assessment Questions
Question No. Answer
1 A
2 B
3 B
4 D
5 C
6 C
7 A
8 A
9 C
10 D
11 D
12 A
13 B
14 A
15 B
16 A
17 B
18 C
19 B
20 B
Activity
Activity type: Online/offline Duration: 50 min
1. Explain the truth table of 8x1 the multiplexer using the logic circuit.
Bibliography
● Mano, M. (1992). Computer System Architecture. PHI.
● Stallings, W. (2015). Computer System Architecture. PHI.
● Brown, S., & Vranesic, Z. (2008). Fundamentals of Digital Logic with Verilog Design (2nd
ed.). McGraw-Hill.
● Mano, M. (2017). Digital Logic and Computer Design. Pearson.
● Jain, R. P. (2009). Modern Digital Electronics (4th ed.). Tata McGraw Hill.
e-References
● https://www.elprocus.com/what-is-multiplexer-and-demultiplexer-types-and-differences/
Video links
● https://youtu.be/t3Ed13z9uz8
● https://youtu.be/FKvnmxte98A
● https://www.javatpoint.com/multiplexer-digital-electronics
● https://www.javatpoint.com/multiplexer-digital-electronics
● https://www.javatpoint.com/multiplexer-digital-electronics
● https://www.javatpoint.com/multiplexer-digital-electronics
● https://www.geeksforgeeks.org/multiplexers-in-digital-logic/
● https://www.geeksforgeeks.org/multiplexers-in-digital-logic/
● https://www.geeksforgeeks.org/multiplexers-in-digital-logic/
● https://www.geeksforgeeks.org/multiplexers-in-digital-logic/
● https://www.geeksforgeeks.org/multiplexers-in-digital-logic/
● https://www.geeksforgeeks.org/multiplexers-in-digital-logic/
● https://www.geeksforgeeks.org/multiplexers-in-digital-logic/
● https://www.geeksforgeeks.org/multiplexers-in-digital-logic/
Fig.13: Block diagram of SOP Logic using 4x1 multiplexer [AB as Select]
● https://www.geeksforgeeks.org/multiplexers-in-digital-logic/
Fig.14: Block diagram of SOP Logic using 4x1 multiplexer [AC as Select]
● https://www.geeksforgeeks.org/multiplexers-in-digital-logic/
● https://www.geeksforgeeks.org/multiplexers-in-digital-logic/
● https://www.javatpoint.com/de-multiplexer-digital-electronics
● https://www.javatpoint.com/de-multiplexer-digital-electronics
● https://www.javatpoint.com/de-multiplexer-digital-electronics
● https://www.javatpoint.com/de-multiplexer-digital-electronics
Keywords
● Multiplexer
● De-multiplexer
● Truth Table
● Boolean Equation
● Logic Circuit
Module - 4
Unit - 4
Sequential Circuits
Flip Flops
Aim ---------------------------------------------------------------------------------------------------- 242
Instructional Objectives -------------------------------------------------------------------------- 242
Learning Outcomes ------------------------------------------------------------------------------- 242
4.4 Sequential Circuits ---------------------------------------------------------------------------- 243
4.4.1 SR Flip Flops --------------------------------------------------------------------------------- 244
Self-Assessment Questions -------------------------------------------------------------- 247
4.4.2 JK Flip Flop ----------------------------------------------------------------------------------- 249
Self-Assessment Questions -------------------------------------------------------------- 251
4.4.3 D Flip Flop ------------------------------------------------------------------------------------ 253
Self-Assessment Questions -------------------------------------------------------------- 255
4.4.4 T Flip Flop ------------------------------------------------------------------------------------ 257
Self-Assessment Questions -------------------------------------------------------------- 259
4.4.5 Master Slave JK Flip Flop ---------------------------------------------------------------- 261
Self-Assessment Questions -------------------------------------------------------------- 263
Summary --------------------------------------------------------------------------------------------- 265
Terminal Questions ------------------------------------------------------------------------------- 266
Answer Keys ----------------------------------------------------------------------------------------- 267
Activity ---------------------------------------------------------------------------------------------- 268
Glossary --------------------------------------------------------------------------------------------- 269
Bibliography ---------------------------------------------------------------------------------------- 269
e-References -------------------------------------------------------------------------------------- 269
Video Links ---------------------------------------------------------------------------------------- 270
Image Credits -------------------------------------------------------------------------------------- 270
Keywords -------------------------------------------------------------------------------------------- 271
Instructional Objectives
This unit intends to:
● Describe the design of sequential logic circuit – Flip Flops.
● Explain the Flip-Flops function.
● Demonstrate why sequential logic circuits are important in various computer applications.
Learning Outcomes
Upon completion of this unit, you will be able to:
● Identify the basic components and applications of an architectural and logic design of
sequential logic circuit- Flip Flops.
● Summarise various flip-flop functions.
● Analyse why the architectural and logic design of sequential logic circuits are essential
in digital system.
The clock signals are not used by the Asynchronous sequential circuits. The asynchronous circuit
is operated through the pulses. So, the changes in the input can change the state of the circuit. The
asynchronous circuits do not use clock pulses. The internal state is changed when the input variable
is changed. The un-clocked flip-flops or time-delayed are the memory elements of asynchronous
sequential circuits. The asynchronous sequential circuit is similar to the combinational circuits with
feedback.
● S-R flip-flop
● J-K flip-flop
● D flip-flop
● T flip-flop
1) SR Flip Flop:
This is the simplest flip-flop circuit. It has a set input (S) and a reset input (R). When in this circuit when
S is set as active, the output Q would be high and the Q’ will be low. If R is set to active, then the output
Q is low and the Q’ is high. Once the outputs are established, the results of the circuit are maintained
until S or R get changed, or the power is turned off.
Q(t+1) = S+R’Q(t)
A) Present input
B) Past input
C) Present input and present state
D) None of the above
A) Set Reset
B) Simple-Reset
C) Single-Reset
D) None of the above
3) When the set is enabled in S-R flip flop then the output will be __________.
A) Reset
B) Set
C) No change
D) Indeterminate
4) When both set and reset are disabled in S-R flip flop then the output will be __________.
A) Set
B) Reset
C) No change
D) Indeterminate
5) A basic S-R flip-flop can be constructed by cross-coupling of which basic logic gates.
A) AND or OR gates
B) XOR or XNOR gates
C) NOR or NAND gates
D) AND or NOR gates
6) The truth table for an S-R flip-flop has how many VALID entries.
A) 4
B) 3
C) 2
D) 1
7) The circuits of NOR based S-R latch classified as asynchronous sequential circuits, why?
A) One
B) Two
C) Three
D) Four
A) Registers
B) Counters
C) Storage devices
D) All of the above
A) Combinational
B) Sequential
C) Both a and b
D) None of the above
Q(t+1) = JKQ’(t)+R’Q(t)
A) Temporary
B) Random
C) Nonrandom
D) True
A) One-way
B) Two ways
C) Three ways
D) Four ways
A) Unstable
B) Bi-stable
C) Both a and b
D) None of the above
14) The J-K flip flop characteristic similar to _________ flip flop.
A) D flip flop
B) T flip flop
C) S-R flip flop
D) None of the above
A) J=1, K=1
B) J=0, K=0
C) J=1, K=0
D) J=0, K=1
A) J=1, K=1
B) J=0, K=0
C) J=1, K=0
D) J=0, K=1
A) D flip flop
B) T flip flop
C) S-R flip flop
D) None of the above
A) SR
B) JK
C) T
D) Both SR and JK
A) Binary inputs
B) Clock signal
C) Both a and b
D) None of the above
Q(t+1) = D
A) Distant
B) Data
C) Desired
D) Delay
A) 1
B) 2
C) 3
D) 4
A) S-R
B) J-K
C) T
D) S-K
A) Has no effect
B) Goes high
C) Goes low
D) Has effect
26) Which of the following describes the operation of a positive edge-triggered D flip-flop?
A) Synchronous
B) Asynchronous
C) Both a and b
D) None of the above
A) One-way
B) Two ways
C) Three ways
D) Four ways
29) How many NAND gates does the D flip flop circuit consists of.
A) One
B) Two
C) Three
D) Four
A) The D input is HIGH and the clock transitions from HIGH to LOW
B) The D input is HIGH and the clock transitions from LOW to HIGH
C) The D input is HIGH and the clock is LOW
D) The D input is HIGH and the clock is HIGH
Q(t+1)=T′Q(t)+TQ(t)′=T⊕Q(t)
A) Trigger
B) Toggle
C) Trigger or toggled
D) None of the above
32) How many possible conversions are there to convert T flip flop to other flip flops.
A) One way
B) Two ways
C) Three ways
D) Four ways
A) More power
B) More area
C) Less power
D) Both a and b
A) D flip flop
B) T flip flop
C) S-R flip flop
D) None of the above
A) Flip-flop
B) Latch
C) Strobe
D) Adder
36) Which flip-flop plays a vital role by functioning as the basic building block of a ripple count-
er?
A) S-R flip-flop
B) J-K flip-flop
C) D flip-flop
D) T flip-flop
A) 2
B) 4
C) 2n – 1
D) 4n – 1
38) Which among the following statements is correct for the triggering associated with
bistable elements?
40) Which among the following is not a mode of Flip Flop representation?
A) Characteristic Equations
B) Excitation Tables
C) Finite State Machines (FSM)
D) Variable Entered Mapping (VEM)
However, due to these feedback paths, a new problem is raised in the circuit, which is called race
around condition. Race around condition in the JK flip is a major problem in which the outputs of flip
flop are toggled continuously till the end of applied clock signal.
To avoid the problem of race around condition in JK flip flop, we use the JK flip flop in the Master and
Slave Mode. Hence, the JK flip flop is called Master-Slave Flip Flop.
Master Slave JK Flip Flop is a combination of two JK flip flops which are connected in the cascaded
manner as shown in Figure-1.
In this combination of two JK flip flop, one acts as a master flip flop and the other acts as a slave flip
flop. In this master-slave flip flop, the outputs of the master JK flip flop are connected to the inputs of
the slave JK flip flop. The outputs of the slave flip flop are fed back to the inputs of the master JK flip
flop.
In the master-slave JK flip flop, a NOT gate (Inverter) is also used which is connected to clock signal
in a manner that the inverted clock signal is applied to the slave flip flop.
Therefore, when clock signal to master flip flop is 0, then for slave flip flop the clock signal is 1, and if
the clock signal to master flip flop is 1, then for the slave flip flop it 0.
When the clock pulse goes to high, the slave flip flop becomes inactive and the inputs J and K can
control the state of the system.
When the clock pulse goes back to low, the information is transferred from master flip flop to the slave
flip flop, and the final output of the system is obtained.
From the circuit, it is clear that the master flip flop is positive level triggered and the slave flip flop is
negative level triggered. Consequently, the master flip flop responds before the slave flip flop. Now, let
us discuss the operation of the master-slave JK flip flop for different combinations of inputs J and K.
When J = 0 and K = 0, both JK flip flops remains inactive and hence the output Q remains unchanged.
This is called Hold State of the master-slave JK flip flop.
When J = 0 and K = 1, the output Q’ of the master flip flop is high and goes to the input K of the slave
flip flop. The clock signal forces the slave flip flop to reset. Therefore, the slave flip flop has the same
output has the master flip flop, i.e., high Q’ and low Q. This is called reset state of the master-slave
JK flip flop.
When J = 1 and K = 0, the output Q of the master flip flop is high and goes to the input J of the slave
flip flop, the negative transition of the clock signal sets the slave flip flop. Hence, this is called the set
state of the master-slave JK flip flop.
When J = 1 and K = 1, for this input combination, the master flip flop toggles on the positive transition
of the clock pulse and the slave flip flop toggles on the negative transition of the clock pulse. Hence,
the problem of the race around condition of the JK flip flop is solved.
42) In a positive edge triggered JK flip flop, a low J and low K produces.
A) High state
B) Low state
C) Toggle state
D) No Change State
43) If one wants to design a binary counter, the preferred type of flip-flop is ___________.
A) D type
B) S-R type
C) Latch
D) J-K type
44) Which of the following flip-flops is free from the race around the problem?
A) T flip-flop
B) SR flip-flop
C) Master-Slave Flip-flop
D) D flip-flop
A) S-R flip-flop
B) J-K flip-flop
C) Master slave flip-flop
D) D Flip-flop
46) In JK flip flop same input, i.e., at a particular time or during a clock pulse, the output will
oscillate back and forth between 0 and 1. At the end of the clock pulse the value of output Q
is uncertain. The situation is referred to as:
A) Conversion condition
B) Race around condition
C) Lock out state
D) Forbidden State
A) OR Gate
B) AND Gate
C) Inverter
D) Full Adder
A) 3
B) 2
C) 4
D) 5
A) Stable devices
B) Astable devices
C) Bistable devices
D) Monostable devices
A) The output changes state only when any of the input is triggered
B) The output changes state only when the clock input is triggered
C) The output changes state only when the input is reversed
D) The output changes state only when the input follows it
3. With the help of a logic circuit and truth table explain the operation of the SR Flip flop.
7. With the help of a logic circuit and truth table explain the operation of JK Flip flop.
11. With the help of a logic circuit and truth table explain the operation of the D Flip flop.
12. Compare the truth table and excitation table of the D flip-flop.
15. With the help of logic circuit and truth table explain the operation of T Flip flop.
Self-Assessment Questions
Question No. Answer
1 C
2 A
3 B
4 C
5 C
6 B
7 C
8 C
9 D
10 A
11 D
12 C
13 B
14 C
15 A
16 B
17 A
18 C
19 D
20 B
21 D
22 A
23 A
24 A
25 A
26 B
27 A
28 C
29 D
30 B
Activity
Activity type: Online/offline Duration: 60 mins
Bibliography
● Mano, M. (1992). Computer System Architecture. PHI.
● Stallings, W. (2015). Computer System Architecture. PHI.
● Brown, S., & Vranesic, Z. (2008). Fundamentals of Digital Logic with Verilog Design (2nd
ed.). McGraw-Hill.
● Mano, M. (2017). Digital Logic and Computer Design. Pearson.
● Jain, R. P. (2009). Modern Digital Electronics (4th ed.). Tata McGraw Hill.
e-References
● https://www.tutorialspoint.com/digital_circuits/digital_circuits_sequential_circuits.htm
● https://www.tutorialspoint.com/digital_circuits/digital_circuits_latches.htm
● https://www.tutorialspoint.com/digital_circuits/digital_circuits_flip_flops.htm
● https://www.tutorialspoint.com/digital_circuit/digital_circuits_conversion_of_flip_flops.htm
● https://www.electronics-tutorials.ws/sequential/seq_2.html
● http://www.pmcgibbon.net/teachcte/electron/degloss1.htm
Image Credits
● https://www.javatpoint.com/sequential-circuits-in-digital-electronics
● https://forums.ni.com/t5/LabVIEW/JK-ff-in-counter-circuit/td-p/1009411
● https://www.ques10.com/p/14914/draw-the-circuit-of-jk-ff-using-nand-gates-and-wri/?
● https://electronics.stackexchange.com/questions/497756/what-is-the-relevance-of-a-q-
in-the-d-flip-flop-when-using-for-a-memory-module
● https://www.javatpoint.com/basics-of-flip-flop-in-digital-electronics
● https://www.javatpoint.com/master-slave-jk-flip-flop-in-digital-electronics
Module - 4
Unit - 5
Conversion of Flip-Flops
Instructional Objectives
This unit intends to:
● Demonstrate the conversion of Flip-Flops to different types
Learning Outcomes
Upon completion of this unit, you will be able to:
● List the several forms of conversion of Flip-Flops and their relevance
Here SR flip-flop is to be designed using JK flip-flop. Thus, one needs to write the truth table for SR
flip-flop.
Step 2: Obtain the Excitation Table for the given Flip-Flop from its Truth Table
Excitation tables provide the details regarding the inputs which must be provided to the flip-flop to ob-
tain a definite next state (Qn+1) from the known current state (Qn).
Table 2: Truth Table for JK Flip-Flop Table Table 3: Excitation Table for JK Flip-Flop
This means that to obtain the next state, Qn+1 as 0 from the current state Qn = 0, J must be made
zero while K can be either 0 or 1.
This is indicated by the first row of the excitation table (blue entries in the first row of excitation table)
where the value of K is expressed as ‘X’ indicating do not care condition. Similarly, to obtain the next
state as 1 from the current state 0, one must have J equal to 1 while K can be either 0 or 1 (indicated
by green entries of the truth table).
This leads to the second row of excitation table (green entries) to be filled with values Qn = 0, Qn+1 =
1, J = 1 and K = X. On the same grounds, the entire excitation table needs to be filled (entries in pink
and dark red colours).
Step 3: Append the Excitation Table of the given Flip-Flop to the Truth Table of the Desired Flip-Flop
appropriately to obtain Conversion Table.
Here the conversion table is obtained by filling-up the values of the J and K inputs for the given Qn
and Qn+1, by referring to the excitation table.
In this case, one needs to arrive at the logical expressions for the inputs J and K in terms of S, R, and
Qn using suitable simplification techniques like K-map.
Step 5: Design the Necessary Circuit and make the Connections accordingly
A. Present input
B. Past input
C. Present input and present state
D. None of the above
A. 2
B. 4
C. 1
D. 3
A. Set Reset
B. Simple-Reset
C. Single-Reset
D. None of the above
A. One
B. Two
C. Three
D. Five
A. SR
B. JK
C. T
D. Both SR and JK
A. SR
B. JK
C. T
D. Both SR and JK
7) How many conversions are there to convert JK flip flop to other flip flops?
A. One
B. Two
C. Three
D. Four
Terminal Questions
1. Convert of T Flip Flop to D Flip Flop.
2. Convert of T Flip Flop to JK Flip Flop.
3. Convert of SR Flip Flop to D Flip Flop.
4. Convert of JK Flip Flop to T Flip Flop.
Answer Keys
Self-Assessment Questions
Question No. Answer
1 C
2 B
3 A
4 D
5 A
6 B
7 B
Glossary
● Q and Q’: The complementary outputs of a flip flop
● Edge-Triggered D Flip Flop: A flip-flop circuit in which the inputs are clocked, and the
outputs appear on the same clock edge.
● Pulse: A sudden change from one level to another, followed a time by a sudden change
back to the original level.
● Truth Table: Method for describing how a logic circuit’s output is dependent upon the
logic levels present at the circuit’s inputs.
Bibliography
● Mano, M. (1992). Computer System Architecture. PHI.
● Stallings, W. (2015). Computer System Architecture. PHI.
● Brown, S., & Vranesic, Z. (2008). Fundamentals of Digital Logic with Verilog Design (2nd
ed.). McGraw-Hill.
● Mano, M. (2017). Digital Logic and Computer Design. Pearson.
● Jain, R. P. (2009). Modern Digital Electronics (4th ed.). Tata McGraw Hill.
e-References
● https://www.tutorialspoint.com/digital_circuit/digital_circuits_conversion_of_flip_flops.htm
● https://www.watelectronics.com/mcq/flip-flops/
Keywords
● Truth Table
● Excitation Table
● Conversion Table
● K-Map
● Circuit Diagram